GENERAL PRACTITIONER EXAM
Question Summary
0 of 100 questions completed
Questions:
- 1
- 2
- 3
- 4
- 5
- 6
- 7
- 8
- 9
- 10
- 11
- 12
- 13
- 14
- 15
- 16
- 17
- 18
- 19
- 20
- 21
- 22
- 23
- 24
- 25
- 26
- 27
- 28
- 29
- 30
- 31
- 32
- 33
- 34
- 35
- 36
- 37
- 38
- 39
- 40
- 41
- 42
- 43
- 44
- 45
- 46
- 47
- 48
- 49
- 50
- 51
- 52
- 53
- 54
- 55
- 56
- 57
- 58
- 59
- 60
- 61
- 62
- 63
- 64
- 65
- 66
- 67
- 68
- 69
- 70
- 71
- 72
- 73
- 74
- 75
- 76
- 77
- 78
- 79
- 80
- 81
- 82
- 83
- 84
- 85
- 86
- 87
- 88
- 89
- 90
- 91
- 92
- 93
- 94
- 95
- 96
- 97
- 98
- 99
- 100
Information
Hi, Welcome to General Practioner Exam
You have already completed the Exam before. Hence you can not start it again.
Exam is loading...
You must sign in or sign up to start the Exam.
You have to finish following Exam, to start this Exam:
Results
0 of 100 questions answered correctly
Your time:
Time has elapsed
You have reached 0 of 0 points, (0)
Average score |
|
Your score |
|
Categories
- Not categorized 0%
Pos. | Name | Entered on | Points | Result |
---|---|---|---|---|
Table is loading | ||||
No data available | ||||
- 1
- 2
- 3
- 4
- 5
- 6
- 7
- 8
- 9
- 10
- 11
- 12
- 13
- 14
- 15
- 16
- 17
- 18
- 19
- 20
- 21
- 22
- 23
- 24
- 25
- 26
- 27
- 28
- 29
- 30
- 31
- 32
- 33
- 34
- 35
- 36
- 37
- 38
- 39
- 40
- 41
- 42
- 43
- 44
- 45
- 46
- 47
- 48
- 49
- 50
- 51
- 52
- 53
- 54
- 55
- 56
- 57
- 58
- 59
- 60
- 61
- 62
- 63
- 64
- 65
- 66
- 67
- 68
- 69
- 70
- 71
- 72
- 73
- 74
- 75
- 76
- 77
- 78
- 79
- 80
- 81
- 82
- 83
- 84
- 85
- 86
- 87
- 88
- 89
- 90
- 91
- 92
- 93
- 94
- 95
- 96
- 97
- 98
- 99
- 100
- Unanswered
- Answered
- Review
-
Question 1 of 100
1. Question
1 pointsWhich of the following statements concerning the thymus is true?
Correct
Incorrect
Explanation:
Cortical thymocytes are immature forms, and either do not express CD4 or CD8 (double negative cells) or express both CD4 and CD8 (double positive cells). As the cells mature, they pass to the thymic medulla where they lose expression of either CD4 or CD8, to become single positive cells.
Negative selection occurs at the stage when thymocytes express both CD4 and CD8, but co-expression of these markers does not mediate negative selection. Negative selection occurs when a thymocyte expresses a TcR with high affinity for self-antigen: MHC complexes in the thymic micro-environment.
Once a thymocyte has successfully rearranged and expressed an -alpha/beta or gamma/delta TcR it is committed to that lineage.
Thymocytes whose TcR bind with high affinity to self Ag/MHC complexes are clonally deleted by a process of negative selection. B cells express IgM and IgD; T cells do not! -
Question 2 of 100
2. Question
1 pointsAn old man who lives alone and prepares his own food is found to have numerous ecchymotic hemorrhage areas around his fragmented hair follicles. Hematomas are also found in limb muscles. Except for the absence of teeth, the examination is normal. PT, APTT and full blood count is normal except for a hematocrit of 28%. What deficiency is present?
Correct
Incorrect
Explanation:
This man has features of scurvy. Scurvy is the clinical state arising from dietary deficiency of vitamin C (ascorbic acid). It results in impaired collagen synthesis. The typical pathological manifestations of vitamin C deficiency are noted in dentine, osteoid and capillary vessel wall tissues (Clinical features include gum swelling, friability, bleeding, and infection with loose teeth; mucosal petechiae; scleral icterus (late, probably secondary to hemolysis); and pale conjunctiva are seen. Fractures, dislocations, and tenderness of bones are common in children. Bleeding into muscles and joints may be seen. Perifollicular hyperkeratotic papules, perifollicular hemorrhages, purpura, and ecchymoses are the classical skin manifestation of scurvy
-
Question 3 of 100
3. Question
1 pointsWhich of the following is true of breast cancer?
Correct
Incorrect
Explanation:
Autosomal dominant inheritance is the main means of inheritance, and the BRCA1 and BRCA2 mutations are two such genes.
An estimated 5-10% of breast cancers are inherited. BRCA1 is also associated with increased risk of pancreatic cancer, and BRCA2 is associated with pancreatic cancer, prostate cancer and melanoma. It is important to remember that inheritance of these mutations leads to an increased risk of developing a malignancy, not the certainty of developing a malignancy. -
Question 4 of 100
4. Question
1 pointsTwenty of thirty in an adult ward develop colicky abdominal pain and diarrhoea without vomiting between 21:00 and 01:00 hrs. Meat stew was served for lunch at noon. Which of the following is the likely diagnosis?
Correct
Incorrect
Explanation:
This food poisoning with no vomiting and an incubation period between 9-13 hours is typical of Clostridium perfringens. The history is too long for a typical Staph, aureus infection (vomiting a typical feature, incubation period 1-6 hours) and rather short of enterovirus (24 hours).
The predominant symptom of B.cereus (incubation period 1-5 hours) is marked vomiting with diarrhoea occasionally seen.
Most infections with C. neoformans consist of a lung infection and not diarrhea.
Rotavirus is the most common cause of severe diarrhoea among infants and young children.
Supportive treatment is all that is generally required with symptoms resolving after 24 hours. -
Question 5 of 100
5. Question
1 pointsWhat tumor marker is most likely to be elevated in pheochromocytoma?
Correct
Incorrect
Explanation:
Urine metanephrine testing is primarily used to help detect and rule out pheochromocytomas in symptomatic patients. It may also be ordered to help monitor the effectiveness of treatment when a pheochromocytoma is discovered and removed and to monitor for recurrence. The alpha-fetoprotein (AFP) tumor marker may be elevated in HCC. Prostate specific antigen (PSA) may rise in prostate carcinoma. CA- 125 can be elevated in ovarian carcinoma. Carcinoembryonic antigen (CEA) may be elevated in colon carcinoma.
-
Question 6 of 100
6. Question
1 pointsWhich of the following is NOT a characteristic of Diverticula?
Correct
Incorrect
Explanation:
Diverticula occur most commonly in the sigmoid colon and decrease in frequency in the proximal colon. Congenital diverticula are herniations of the entire thickness of the intestinal wall. By contrast, acquired diverticula are herniations of the mucosa through the muscularis, usually at the site where the nutrient artery penetrates the muscularis. Both increased intraluminal pressure and decreased fiber intake have been postulated to predispose to the development of diverticula.
-
Question 7 of 100
7. Question
1 pointsVirchow´s criterion is used to describe the optimal conditions for forming thrombi. All of the following are considered part of Virchow´s criterion, EXCEPT
Correct
Incorrect
Explanation:
Virchow´s criterion is a triad of stasis, endothelial damage, and hypercoagulability. Fibrinolysis is not included in the triad of Virchow.
-
Question 8 of 100
8. Question
1 pointsA 20 year old intravenous drug user presents to the emergency room with fever, palpitation, and shortness of breath. On physical examination she has a systolic murmur, Roth´s spots and splinter hemorrhages. Her ESR is 120. The most common infectious organism which will grow from her blood cultures is Correct
Incorrect
Explanation:
Staphylococcus aureus is an aerobic Gram positive cocci which causes infective endocarditis most commonly in intravenous drug users and usually affects the tricuspid valve, causing valvular infected vegetations and, eventually, tricuspid regurgitation. Candida albicans is a fungal organism which can cause endocarditis in immunocompromised patients. It is not common. Klebsiella is a Gram negative organism which does not usually cause endocarditis, but does commonly cause pneumonia in alcoholics. Bacteroides is an anaerobic organism which is most commonly located in the bowels and causes bowel infection. Tuberculosis very rarely causes endocarditis, but does affect the pulmonary system.
-
Question 9 of 100
9. Question
1 pointsA 21 year student presents with dysmenorrhea and lower abdominal tenderness. Pelvic examination shows adnexal tenderness and mucopurulent discharge without bleeding. A scraping from the cervix is stained with Giemsa stain shows intracytoplasmic inclusion bodies in the infected epithelial cells. Unusual statement about the biochemical makeup of this organism is
Correct
Incorrect
Explanation:
This young woman is infected with Chlamydia trachomatis. This genus of organisms is unusual in that they do not make ATP and are thus obligate intracellular parasites, and they possess a cell wall in which muramic acid is missing from the peptidoglycan. Thus they have a cell wall, but it is made of highly modified peptidoglycan (it can be called peptidoglycan at all). It has sterols in its membrane is a statement that would describe the mycoplasmas. These are the only prokaryotes that totally lack a cell wall, but instead have sterols in their membranes. It lacks a cell wall would be a statement that describes the mycoplasma. These prokaryotes are devoid of a cell wall, but have sterols in their membranes. Its cell membrane contains ergosterol would describe the fungi. This molecule is unique to fungi and is the target molecule for nystatin and the imidazoles. A cell wall containing mycolic acids describes the genus Mycobacterium. Mycolic acids are long chain fatty acids that give this group its resistance to environmental stresses and its acid fast staining characteristics.
-
Question 10 of 100
10. Question
1 pointsA 7 year old child develops edema, hypertension and hematuria several weeks after a severe streptococcal sore throat. Best description of the properties of the most likely infectious organism is given by which one of the following?
Correct
Incorrect
Explanation:
The most likely cause of glomerulonephritis following sore throat is Streptococcus pyogenes. This is a Group A streptococcus, which is characterized by beta hemolysis and inhibition by the antibiotic bacitracin. Streptococcus pyogenes pharyngitis can also be followed by rheumatic fever. Streptococcus pneumoniae is an example of an alpha hemolytic streptococcus that is inhibited by optochin and lysed by bile. Streptococcus mutans is an example of an alpha hemolytic streptococcus that is not inhibited by Optochin nor lysed by bile. Enterococcus faecalis is an example of a beta hemolytic streptococcus that can grow in 40% bile and 6.5% sodium chloride. Streptococcus agalactiae is an example of a beta hemolytic streptococcus that is resistant to bacitracin and positive for the CAMP test (an incomplete hemolysin).
-
Question 11 of 100
11. Question
1 pointsA 25 year old male is infected with HIV during a sexual encounter with a prostitute. In the period immediately after the encounter he does not notice any symptom. During the asymptomatic latent phase of his infection, the virus is actively proliferating and can be found in association with which of the following?
Correct
Incorrect
Explanation:
Follicular dendritic cells in the germinal centers of lymph nodes are important reservoirs of HIV. Although some follicular dendritic cells are infected with HIV, most viral particles are found on the surface of their dendritic processes. Follicular dendritic cells have receptors to the Fc portion of immunoglobulins that serve to trap HIV virions coated with anti-HIV antibodies. These coated HIV particles retain the ability to infect CD4+ T cells as they traverse the dendritic cells. B lymphocytes have a surface marker (CD21 protein-a complement receptor) to which an Epstein-Barr envelope glycoprotein can bind. The virus associates with the host cell genome, producing a latent infection. These B cells undergo polyclonal activation and proliferation. Ganglion cells, particularly the satellite cells around the ganglion cells in the dorsal root ganglia, can be infected by varicella zoster. Herpes type I and II infect neurons that innervate skin and mucous membranes. Oligodendrocytes are directly infected by two viruses: JC virus (a polyomavirus) and measles virus. JC virus causes progressive multifocal
leukoencephalopathy (PML); and measles (virus produces a latent syndrome, called subacute sclerosing panencephalitis (SSPE). Peripheral nerves are indirectly affected by HIV virus in the AIDS associated myopathy. The disease is characterized by a subacute onset of proximal muscle weakness sometimes with pain and elevated levels of creatine kinase. The muscles and nerves are infiltrated with mononuclear cells including HIV positive macrophages. -
Question 12 of 100
12. Question
1 pointsA 4 year old boy presents with epistaxis. He has a history of multiple bacterial and viral respiratory tract infections and eczema. His uncle had similar problems. Examination is remarkable for multiple petechial lesions on the skin and mucous membranes. Serum IgE is increased and platelets are decreased. Cytokine that is most likely to be elevated in this patient is
Correct
Incorrect
Explanation:
This patient has Wiskott Aldrich syndrome, which is an X-linked condition characterized by eczema, thrombocytopenia, and repeated infections. Affected children may present with bleeding and often succumb to complications of bleeding infection, or lymphoreticular malignancy. The platelets are small, have a shortened half life, and appear to be deficient in surface sialophorin (CD43). Splenectomy can correct the thrombocytopenia but not the immune defect. Serum IgM is usually decreased, whereas lgE is frequently increased. Mutations in the Wiskott-Aldrich serum protein (WASP) gene on the short arm of the X chromosome are responsible for this disease. The TH2 cytokine that is most closely linked with isotype switching to promote IgE synthesis is IL-4. IL-1 is also known as endogenous pyrogen. It is a product of macrophages and it acts to set the hypothalamic temperature set point. IL-2 is a cytokine produced by all T-helper lymphocytes that causes cell division in virtually all lymphocytes. IL-3 is a product of many cells that stimulates bone- marrow production of myeloid lineage cells. IL-5 is a TH2 cytokine that stimulates B cells to isotype-switch to making IgA.
-
Question 13 of 100
13. Question
1 pointsA 27 year old woman returned from Malaysia. She presents with fever, chills, sweats, headache, and body aches that occurred every other day despite acetaminophen and over the counter flu medications. She is prescribed chloroquine as treatment after complete workup. The feature of the pathogen´s live cycle caused her fever to repeat at 48-hour intervals is
Correct
Incorrect
Explanation:
A cyclic pattern of fever is the hallmark of malaria caused by Plasmodium spp. protozoan parasites that are transmitted by mosquito bite. The fever pattern results from the synchronous, repetitive lysis of erythrocytes by the parasite as it undergoes the process of erythrocytic schizogony. This is the process of fission inside the red blood cell is that eventually swells the cell with daughter merozoites to the point that the cell bursts and releases pyrogenic materials into the bloodstream. The fever spikes each time a new crop of daughter merozoites is liberated into the bloodstream to infect another generation of cells. Therefore the periodicity of the malarial paroxysm is a reflection of the length of time that it takes for one parasite to grow and divide inside an erythrocyte and eventually lyse it. The species of malaria have different speeds of the erythrocytic schizogonic cycle: P. vivax, P. ovale and P. falciparum all require 48 hours to develop inside and lyse cells; P. maleriae requires 72 hours. There fore, vivax, ovale, and falciparum malarias are referred to as being tertian malarias and maleriae is referred to as quartan malaria. Exoerythrocytic schizogony does not associate with any symptoms in this patient. This is the process of fission (schizogony) that occurs in hepatocytes at the beginning of the infection. The number of liver cells lysed during this stage of this cycle is not sufficient to cause symptoms. Gametocyte development occurs late in the erythrocytic stage of malaria and is not associated with any symptoms. Gametocytes are haploid gametes that must be ingested by a mosquito to complete the sexual cycle of the parasite.
Sporozoite cell invasion begins the malaria life cycle in humans. Sporozoites are the infectious forms that are injected with the mosquito saliva. They go immediately to the liver, where they penetrate hepatocytes and multiply by schizogony. There are no symptoms of malaria until the parasites enter the bloodstream and begin to cause the synchronous lysis of large numbers of blood cells. Trophozoite hemoglobin digestion does not by itself cause symptoms, although the release of the indigestible waste (hemozoin, malarial pigment) when erythrocytes are lysed clearly contributes to the fever. Below is a figure that summarizes the Plasmodium life cycle. -
Question 14 of 100
14. Question
1 pointsA 25 year old woman has tremors weight loss and discomfort on warm days. She also complains of abdominal fullness. On examination a palpable abdominal mass is revealed. No ophthalmopathy or neck masses are noted. Ultrasound imaging revealed a unilateral ovarian cystic mass. Serum analysis reveals low TSH levels. The most likely diagnosis is which one of the following?
Correct
Incorrect
Explanation:
The patient appears to have the clinical signs of non Graves hyperthyroidism. Serum TSH levels are low and there are no ophthalmic signs. Monodermal teratomas can develop an appearance of mature thyroid tissue, which is termed struma ovarii. Since there is excessive of thyroid hormone, serum TSH is reduced. Ectopic pregnancies should always be considered in any female of reproductive age presenting with abdominal pain. This patient´s symptoms are more protracted and do not present with abdominal pain. Ultrasound would be an appropriate test to assess for ectopic pregnancies. Mature teratomas contain cellular elements derived from all three germ-cell layers; they are composed of hair, teeth, and sebum very rarely, they can produce thyroid hormone, but this is usually not clinically significant because the amount of thyroid tissue in the usual form of mature teratomas is small. Serous cystadenomas are the most common ovarian cystic lesion. As the name implies, they are filled with a simple fluid and do not produce thyroid hormone. Thecoma fibromas are composed of stromal fibroblasts or lipid-laden cells. Usually these occur in combination; hence, the term fibroma thecoma is used. Patients with these tumors do not produce thyroid hormone.
-
Question 15 of 100
15. Question
1 pointsA brain tumor biopsy of a 42 year old male shows a glial neoplasm consisting of atypical astrocytes with scattered mitoses. Besides mitotic activity, which markers can provide information about neoplasm´s proliferative activity?
Correct
Incorrect
Explanation:
Ki-67 is a nuclear factor (of uncertain function) whose expression correlates with neoplastic replicative activity. Its expression can be visualized by immunostaining of formalin fixed, paraffin-embedded sections. Ki-67 labeling correlates with a neoplasm´s rate of growth and, therefore, with prognosis. The bcl 2 gene suppresses apoptosis by different mechanisms. Its abnormal activation is involved in the pathogenesis of low-grade lymphomas but not astrocytomas. GFAP is an intermediate cytoskeletal filament (analogue of keratin and vimentin) expressed exclusively by certain types of glial cells, e.g. astrocytes and ependymal cells. Immunohistochemistry for GFAP is mod diagnostically to confirm an astrocytic origin of a neoplasm, but gives no information about mitotic or proliferative rate. The gene p53 encodes a protein that blocks the cell cycle when damage to DNA occurs. If the damage is successfully repaired, p53 allows the cell cycle to resume; if not, p53 induces apoptosis, thus eliminating dangerous DNA mutations. Mutations of p53 have been found in the great majority of human neoplasms, including gliomas. However, its expression gives no information concerning neoplastic replicative activity. Ubiquitin is a low molecular weight heat shock protein. Its function is to tag aberrant proteins for degradation. It is present in many abnormal intraneuronal inclusions associated with neurodegenerative disorders, such as Lewy bodies, Pick bodies, and neurofibrillary tangles. It has no relationship with mitotic activity or growth rate.
-
Question 16 of 100
16. Question
1 pointsA 65 year old woman has a long standing dementing disorder characterized by deterioration in personality, neglect of personal hygiene, impaired judgment, and disinhibited behavior. MRI shows severe cortical atrophy limited to the frontal lobes and anterior two thirds of the temporal lobes while the remaining cortex is preserved. No evidence of recent or remote infarcts is found. What is the likely diagnosis?
Correct
Incorrect
Explanation:
Not all dementing disorders manifest with the same clinical features. Although there is considerable overlap in clinical symptomatology among different types of dementias, making clinical diagnosis somewhat problematic, there are classic presentations that allow identification of a specific form of dementia with a high degree of confidence. In this case the patient has symptoms due to frontal lobe damage, e.g. disinhibition, impaired judgment, and personality changes. Furthermore, MRI demonstrates a specific pattern of cortical atrophy, restricted to the frontal lobes and anterior portion of the temporal lobes. This combination points toward a group of dementias called frontotemporal dementia, the most frequent form of which is Pick disease. Other forms of frontotemporal dementia are very infrequent. Remember: frontal symptoms in conjunction with frontotemporal atrophy frontotemporal dementia/Pick disease.
Alzheimer disease is the most frequent form of dementia in industrialized countries. Although symptoms due to frontal damage may be present in Alzheimer disease, they are usually associated with a more generalized impairment of higher neurologic functions, e.g., language, memory, and learned movements. In addition, cortical atrophy in Alzheimer disease is widespread and not limited to the frontal and anterior temporal lobes. Creutzfeldt Jacob disease represents the prototype of prion diseases. Cortical atrophy is not a prominent feature of Creutzfeldt-Jacob disease, which manifests with personality changes, memory loss, and seizures, leading to death after a rapid clinical course (a few months to 1 year). Dementia with Lewy bodies, also known as diffuse Levy body disease, is one of the most common forms of dementia in Western countries, possibly more common than vascular dementia. It is characterized by widespread formation of Lewy bodies in the substantia nigra, limbic cortex, and subcortical nuclei, such as the basal nucleus of Meynert. Extrapyramidal symptoms similar to Parkinson disease manifest in this form of dementia as a result of degeneration of dopaminergic: pathways. Vascular dementia is an umbrella term encompassing dementing conditions that arise from pathology of large or small cerebral vessels. It manifests with memory loss associated with focal neurologic symptoms depending on the location of damage. MRI would, identify old or recent infarcts, as well as white matter disease. Conditions associated with vascular dementia include the following:
Multi infarct dementia, which is caused by multiple, scattered brain infarcts secondary to atherosclerosis of large arteries of the circle of Willis and/or carotid arteries. Binswanger disease, which involves rarefaction of cerebral white matter and is caused by hypertension-related arteriolosclerosis. Lacunar infarcts, which consist of small (< 1 cm) infarcts in the striatum and thalamus; this condition is related to arteriolosclerosis. -
Question 17 of 100
17. Question
1 pointsA 67 year old man suffers MI due to thrombotic occlusion at the origin of the left circumflex artery. Cardiac catheterization reveals that he has a left dominant coronary circulation. Which areas of the heart have likely suffered ischemic necrosis?
Correct
Incorrect
Explanation:
A right dominant coronary circulation is present when the posterior descending branch originates from the right coronary artery (80% of individuals). On the contrary the posterior descending artery originates from the left circumflex artery in a left, dominant circulation (20% of individuals). The posterior descending branch gives blood to the posterior half of the interventricular septum. Occlusion of the left circumflex artery in a left dominant circulation will therefore lead to ischemic necrosis in the left ventricular wall and the posterior interventricular septum. The apex of the left ventricle is dependent on the anterior descending branch; thus, occlusion of the left circumflex does not affect this portion of the left ventricle. Infarction of the lateral (free) wall alone will result from occlusion of the circumflex in a right dominant circulation. An isolated infarction of the posterior interventricular septum arises from occlusion of the posterior descending branch. Isolated infarctions of the right ventricular wall are very are and would be caused by occlusion of branches of the right coronary artery.
-
Question 18 of 100
18. Question
1 pointsA boy aged 19 years presents with pneumonia. He has had recurrent pneumonia and sinusitis due to Streptococcus pneumoniae and Haemophilus influenza since the age of 6 months. Careful assessment of his immune function would likely reveal abnormal function of which one of the following?
Correct
Incorrect
Explanation:
The symptoms in the question suggest a deficiency known as common variable hypogammaglobulinemia, characterized by very low serum levels of IgG. In this congenital disease, the number of B cells is normal, but their ability to synthesize IgG and the other immunoglobulins is severely com-promised, leading to recurrent streptococcal and Haemophilus infections. The onset of the recurrent infections usually begins between 6 and 12 months of age concurrent with the decreasing levels of maternal IgG in the newborn. Treatment often involves giving IV gamma globulin to reduce the number of infections.
Abnormal function of macrophages such as that seen in chronic granulomatous disease results in recurrent infections with catalase positive organisms. Streptococcus is a catalase-negative genus. Abnormal function of NK cells is unlikely to present as recurrent bacterial infections, because these cells are primarily involved in cellular, rather than humoral, immunity. Platelets are not directly involved in the immune response. Deficiencies in platelets lead to problems in clotting and produce bleeding disorders. T-cell deficiencies result in severe viral, fungal, and protozoal infections, rather than recurrent bacterial infections. -
Question 19 of 100
19. Question
1 pointsA 37 year old woman has cold and painful fingertips, as well as difficulty swallowing and indigestion. Exam shows a thickened, shiny epidermis over the entire body, with restricted movement of the extremities, particularly the fingers, which appear claw like. Which autoantibodies will likely be found in her serum?
Correct
Incorrect
Explanation:
This patient has systemic sclerosis, also called scleroderma. Antibodies to topoisomerase I (anti-Scl-70) occur in up to 70% of patients with diffuse systemic sclerosis, but only rarely in other disorders. Systemic sclerosis, is characterized initially by excessive fibrosis and edema of the skin, especially the hands and fingers, producing sclerodactyly (characteristic changes in the fingers, which resemble claws). Raynaud phenomenon is common. The diffuse type of systemic sclerosis generally spreads to include visceral organs, such as the esophagus (producing dysphagia), the lungs (producing pulmonary fibrosis), the heart (leading to heart failure or arrhythmia), and the kidneys (renal failure causes 50% of scleroderma deaths). Females are affected more than males (3:1 ratio). A more restricted variant of systemic sclerosis, with a somewhat more benign course, is CREST syndrome (Calcinosis, Raynaud syndrome, Esophageal dysmotility, Sclerodactyly, and Telangiectasia), characterized by the presence of anti-centromere antibodies (although 10% of CREST patients will have anti-topoisomerase antibody as well). Anti-ds DNA and anti-Sm (Smith antigen) are characteristic of systemic lupus erythematosus (SLE) but are not common in patients with systemic sclerosis. Rheumatoid factor is an autoantibody directed against IgG. It is found in patients with rheumatoid arthritis. Anti-SS-A is typically seen in Sjögren syndrome (although it may also be seen in SLE).
-
Question 20 of 100
20. Question
1 pointsA female Anopheles mosquito injects a single sporozoite of Plasmodium vivax into the bloodstream of a susceptible human host. In the liver schizogony requires 10 days and produces 100 merozoites per hepatocyte. Schizogony in the bloodstream produces 10 merozoites per erythrocyte. In 22 days of infection how many meroziotes would be produced?
Correct
Incorrect
Explanation:
This simple mathematical problem asks if the student understands the life cycle of the malaria parasite, specifically Plasmodium vivax. To answer the question, the student needs to understand the terminology and sequence of steps that this agent of “benign tertian” malaria goes through in the human host. The infectious form inoculated by the vector mosquito is the sporozoite. It immediately seeks out a hepatocyte, penetrates it, and undergoes the process of schizogony (asexual fission). The question stem tells us that one sporozoite is injected and within 10 days, 100 daughter merozoites will be produced from each sporozoite. Thus, at 10 days, 100 exoerythroeytic merozoites will be released. These parasitic forms will now seek out erythrocytes, and at 2 day intervals (for P. vivax and P. ovule) will again undergo schizogony; this time, the question tells us to produce 10 merozoites per erythrocyte. There is no repetition of schizogony in the liver, but the cycle is repetitive in the erythrocytes, so that each crop of merozoites will repenetrate erythrocytes and undergo schizogony at 2 day intervals until the immune response or drug treatment stop the cycle. Thus, every 2 days after leaving the liver, there will be ten times the number of merozoites released as were released in the last cycle. The number of merozoites at 22 days will be 100 (day 10) x 10 (day 12) x 10 (day 14) x 10 (day 16) x 10 (day 18) x 10 (day 20) x 10 (day 22), or 100 x 106 or 1 x 108. If this question were posed for P. malariae, the erythrocytic schizogonic cycles would our at 3 day intervals, and if the question were posed for P falciparum, it would not be answerable because asynchronous schizogony and multiple infections of individual erythrocytes can alter the speed of multiplication.
-
Question 21 of 100
21. Question
1 pointsAn 11 year old previously healthy girl develops a gastrointestinal infection with cramping and watery stools. Several days later she begins to pass blood per rectum. Her BUN is rising. CBC reveals anemia and thrombocytopenia, and the peripheral smear is remarkable for fragmented red cells (schistocytes). Infection with which bacterial genera is likely responsible for this syndrome?
Correct
Incorrect
Explanation:
This patient has hemolytic uremic syndrome (HUS), a complication of the Shiga toxin or Shiga like toxin (exotoxins released by Shigella species and the enterohemorrhagic Escherichia coli). In children, HUS usually develops after a gastrointestinal or flu like illness and is characterized by bleeding, oliguria, hematuria, and microangiopathic hemolytic anemia. Presumably the Shiga toxin is toxic to the microvasculature, producing microthrombi that consume platelets and RBCs, and may fragment the red cell membrane. The incorrect choices are all bacteria that may produce an enterocolitis but do not elicit HUS.
A long term consequence of Campylobacter infection is a reactive arthritis or full blown Reiter syndrome. Clostridial enterocolitis is produced by Clostridium difficile, a normal inhabitant of the gut that produces pseudomembranous colitis when other gut flora are suppressed by treatment with antibiotics. Salmonella infections are almost all nontyphoid inflammatory diarrhea, producing a simple enterocolitis that may proceed to sepsis in some cases. Typhoid fever (produced by Salmonella typhi and Salmonella paratyphi) causes a protracted illness that progresses over several weeks and includes rash and very high fevers, but not HUS. Vibrio infections produce copious amounts of watery diarrhea. The major risk of cholera and other Vibrio enteritides is shock due to hypovolemia or electrolyte loss. -
Question 22 of 100
22. Question
1 pointsA 37 year old nonsmoker is found to have a 4 cm mass on his left lung during a routine examination. He has no chronic medical conditions. Both of his parents are alive and well. He undergoes surgery, and pathologic examination of the lesion reveals that it is a primary lung cancer. The most likely type of cancer in him is
Correct
Incorrect
Explanation:
Bronchogenic adenocarcinoma is only weakly associated with smoking. This disorder is the most common lung cancer seen in nonsmokers and in women. Bronchioloalveolar carcinoma is a relatively uncommon form of carcinoma of the lung. It arises in the lung periphery and does not appear to be clearly related to smoking. Large cell (anaplastic) carcinomas are unusual and probably represent undifferentiated variants of squamous cell and bronchogenic adenocarcinomas. Small cell (oat cell) carcinoma is very strongly associated with smoking. Squamous cell carcinoma is very strongly associated with smoking.
-
Question 23 of 100
23. Question
1 pointsA man has bilateral renal failure. His 23 year old sister decides to donate her left kidney to him. Blood flow was 600 mL/min and vascular resistance was 0.16 mm Hg/ mL/min in her kidney before it was removed. Following removal of her kidney what would be increased in her?
Correct
Incorrect
Explanation:
The various organs of the body are arranged in parallel, and therefore contribute parallel resistance to the peripheral circulation. You should recall that adding resistances (R1, R2, R3…) in parallel reduces the total resistance (RT) of a circuit as follows (1/RT = 1/R1 + 1/R2 +1/R3…) so that removing a parallel resistance (R1, R2, or R3) increases the total resistance (RT). For this reason, the total peripheral resistance increases when a kidney is removed. Another way to think about the problem is the following: each kidney provides a pathway of blood to flow from the aorta to the vena cava. When a kidney is removed, there is one less pathway through which blood can flow from the aorta to the vena cava, which means that the resistance to blood flow from the aorta to the vena cava (i.e. the total peripheral resistance) must be increased. Similar logic can be applied to any organ of the body. Removing a kidney should have not have a lasting effect on arterial pressure assuming that the remaining kidney functions normally. The cardiac output decreases when a kidney is removed. The pulmonary blood flow which is equal to the cardiac output of the right heart, should decrease when a kidney is removed. The total renal blood flow will decrease when a kidney is removed.
-
Question 24 of 100
24. Question
1 pointsA 61 year old man has a 25 year history of alcoholism and liver disease. He presents with pain and swelling of his legs. A decrease in which of the following is the likely cause of the peripheral edema?
Correct
Incorrect
Explanation:
The plasma colloid osmotic pressure is often low in alcoholics with chronic liver disease (cirrhosis). The diseased liver cannot produce adequate amounts of albumin, which leads to a decrease in the concentration of albumin in the plasma, i.e., hypoalbuminemia. Because about 75% of the plasma colloid osmotic pressure can be attributed to the presence of albumin in the plasma, the decrease in plasma albumin concentration that occurs in the latter stages of cirrhosis often leads to peripheral edema. Cirrhosis also causes excess fluid to accumulate in the peritoneal cavity as ascites. In the case of ascites the edema results not only from hypoalbuminemia, but also from portal vein obstruction (which increases capillary hydrostatic pressure) as well as the obstruction of lymphatic drainage of the liver. In fact, ascites is observed more often than peripheral edema in liver disease. A decrease in capillary hydrostatic pressure would tend to decrease fluid loss from the capillaries, and thereby oppose the development of edema. A decrease in the colloid osmotic pressure of the interstitial fluid would decrease fluid loss from the capillaries, thereby opposing the development of edema. A decrease in interstitial hydrostatic pressure would tend to increase fluid loss from the capillaries, but this cannot be considered a primary cause of edema because the interstitial hydrostatic pressure actually increases when a tissue becomes edematous. A decrease in precapillary arteriolar resistance which means arteriolar dilation, would increase capillary hydrostatic pressure and tend to cause edema. However, decreased precapillary arteriolar resistance in the peripheral vasculature is not associated with cirrhosis.
-
Question 25 of 100
25. Question
1 pointsA child aged 4 years with chronic tonsillitis presents with a swollen cervical lymph node. What would be most likely revealed on the biopsy of the node
Correct
Incorrect
Explanation:
Chronic nonspecific lymphadenitis is a reactive lymph node hyperplasia that can take three distinct forms, seen in different clinical settings. In chronic bacterial infections, such as this patient´s chronic tonsillitis, there is a stimulation of B cells that leads to prominence of germinal centers. Acute nonspecific lymphadenitis, characterized by necrotic debris, prominent follicles, and, sometimes, a neutrophilic infiltrate, is seen as a response to acute, rather than chronic, infection. This patient has a reactive, rather than neoplastic, etiology for lymph node enlargement. Immunologic reactions induced by drugs such as phenytoin (Dilantin) produce reactive changes in the paracortical T cells, leading to expansion of cells outside the germinal centers (paracortical lymphoid hyperplasia). Sinus histiocytosis is seen in lymph nodes draining cancers (notably breast) and consists of lymphatic sinusoids packed with histiocytes.
-
Question 26 of 100
26. Question
1 pointsA 37 year old female has a recent onset of chest pain. Her BP is 150/60 mm Hg, and auscultation reveals a diastolic murmur along the left sternal border. Lab studies for FTA-ABS test are positive. Which pathologic alteration is likely associated with this presentation?
Correct
Incorrect
Explanation:
This patient has syphilis. In its tertiary stage, syphilis may affect the ascending aorta causing an obliterative endarteritis of the vasa vasorum. This results in scarring of the media with wrinkling of the intima, producing the characteristic tree barking appearance. The aortic ring becomes dilated and the coronary ostia narrowed, leading to aortic regurgitation and myocardial ischemia. Hence, the murmur and the chest pain. Note the typically high systolic pressure and low diastolic pressure accompanying aortic insufficiency. Calcification of aortic cusps with stenosis of the aortic orifice is found in elderly patients with clinically evident aortic stenosis. This would lead to an ejection systolic murmur.
Fibrous thickening of mitral leaflets with commissural fusion is characteristic of chronic rheumatic heart disease and is associated with mitral stenosis. A diastolic murmur is heard at the apex, but this is not associated with a wide difference between systolic and diastolic pressure. Intrachordal ballooning of mitral valve leaflets refers to the ballooning of mitral valve leaflets secondary to mitral valve prolapse. This condition may cause mitral valve regurgitation with a systolic murmur. Severe atherosclerosis of coronary arteries would be highly unusual in a young patient and would not explain signs of aortic insufficiency. However, cardiovascular syphilis leads to development of secondary atherosclerosis of the ascending aorta. -
Question 27 of 100
27. Question
1 pointsA 31 year old man presents to the dermatologist with silvery, scaling plaques on his elbows and knees. His mother suffered the same condition in the past. The most likely diagnosis is which one of the following?
Correct
Incorrect
Explanation:
Psoriasis vulgaris usually appears on the nails, knees, elbows, and scalp. It does not generally affect the mucous membranes. Lesions are well demarcated, coral colored plaques with a white or silvery scale (classic clue). Histologically, epidermal hyperplasia causing thickening and lengthening of the rete ridges is apparent, as is thinning of the epidermis over the dermal papillae. There is a recognized genetic component to this condition. Peak incidence is at age 30. Acne rosacea affects the central face. Erythema, telangiectasias, acneform lesions (papules, cysts, pustules), and rhinophyma (telangiectasias and hyperplasia of nasal soft tissue) are found in various combinations. It is common from ages 30 to 50. Women are affected three times more frequently than men, but the syndrome is more severe in the latter. Acne vulgaris causes comedones, papules, and cysts. It may be related to hormones, drugs, diet, irritants, and genetic factors. Allergy to Propionibacterium acnes has been found to contribute to this condition. Pemphigus vulgaris starts with small vesicles, usually on the oral or nasal mucosa, then spreads to other parts of the body. Bullae are delicate and flaccid. The condition is due to autoantibodies to intercellular junctions between keratinocytes. Nikolsky´s sign (production of blistering by light stroking or rubbing of the skin) is positive. Pemphigus is most common from ages 40 to 60. Pityriasis rosea presents first with a red, scaling, “herald patch” approximately 4 cm in diameter. It is followed within days by eruption in a “short sleeve turtle neck” distribution. The classic clue to the diagnosis is the appearance of crops of small, pink, oval patches in a “fir tree configuration” on the flexural lines. -
Question 28 of 100
28. Question
1 pointsLeast true statement regarding IgA nephropathy is which one of the following?
Correct
Incorrect
Explanation:
IgA nephropathy (Berger´s disease) is the most common glomerulonephritis worldwide, and characteristically affects young males, presenting with frank haematuria after an episode of pharyngitis. However it may also present with proteinuria, microscopic haematuria, renal failure or hypertension. It is probably part of a Spectrum of disease with Henoch Schoenlein Purpura, which presents with arthritis, rash, abdominal pain and nephritis. In both there are mesangial IgA deposits in the kidney
-
Question 29 of 100
29. Question
1 pointsA 66 year old man is admitted with renal failure and is diagnosed with acute tubular necrosis. A least likely to be the cause of acute tubular necrosis is which one of the following?
Correct
Incorrect
Explanation:
Renal failure from ATN occurs in 25% patients with severe hepatic damage. Accelerated hypertension can cause small vessel obstruction, with proliferative endarteritis of intralobular arteries and fibrinoid necrosis of afferent arterioles and glomerular capillary tuft. Corticosteroid therapy has not been associated with ATN. Other causes of ATN include hypotension, hepatic failure, eclampsia and drugs such as aminoglycosides, Cephalosporins, Cisplatin and Amphotericin.
-
Question 30 of 100
30. Question
1 pointsA 21-year-old woman was referred for investigation of iron deficiency anaemia. Her mother died aged 29 years from colonic carcinoma complicating Peutz-Jegher syndrome. Which is the most likely mode of inheritance of Peutz-Jegher syndrome?
Correct
Incorrect
Explanation:
Peutz-Jegher syndrome is a condition characterised by perioral pigmentation and numerous hamartomas of the bowel. Originally it was assumed that these did not predispose to malignancy but studies now suggest the contrary.
The condition is autosomal dominant. -
Question 31 of 100
31. Question
1 pointsAll of the following are true regarding painful scrotal swelling except
Correct
Incorrect
Explanation:
Scrotal pain occurs in both bacterial and nonbacterial epididymitis. Pain can be severe and is sometimes referred to the abdomen. In bacterial epididymitis, the patient may also have fever, nausea (although not common), or urinary symptoms. Urethral discharge may be present if the cause is urethritis. Physical examination reveals swelling, induration, erythema, and marked tenderness of a portion of or all of the affected epididymis and, sometimes, the adjacent testis.
-
Question 32 of 100
32. Question
1 pointsDamage to the gastric mucosa by Helicobacter pylori is facilitated by all of the following EXCEPT
Correct
Incorrect
Explanation:
Monocytes are activated to express HLA-DR and interleukin 2 receptors on their cell surfaces and to produce superoxides, interleukin I, and tumor necrosis factor alpha. Helicobacter pylori produce a variety of proteins which facilitate its damaging effects on the gastric mucosa. Urease catalyzes the hydrolysis of urea to form ammonia and carbon dioxide. The resulting alkaline environment protects the organism from gastric acid and prevents gastric colonization by other bacteria. Surface proteins are chemotactic for neutrophils and monocytes. Proteases and phospholipases degrade the glycoprotein-lipid complex of the mucus gel layer, thereby reducing the thickness and viscosity of this protective layer. Adhesin facilitates the attachment of the organism to gastric epithelial cells.
-
Question 33 of 100
33. Question
1 pointsA 28 year old woman with a long history of type 1 diabetes becomes pregnant. She has difficulty controlling her blood glucose during pregnancy and she gives birth to a baby who dies in the first week of life. Autopsy of the baby reveals a severe cardiac malformation. The most likely pathogenesis of the diagnosis is Correct
Incorrect
Explanation:
Maternal diabetes is best known for causing large but immature-age babies. There is also a specific association between maternal diabetes and transposition of the great vessels which is caused by failure of the AP septum to twist. In transposition of the great vessels, the aorta takes off from the anterior part of the right ventricle and the pulmonary trunk takes of from the posterior part of the left ventricle. This produces a complete separation of the systemic and pulmonary circulations. Without surgical correction most affected infants die within the first months of life, although a patent ductus arteriosus, patent foramen ovale, or ventricular septal defect may allow enough mixing of blood to temporarily sustain life. Associate atrial septal defect (choice D) with Down syndrome. There is an incomplete septum between the ventricles and the atria due to the endocardial cushion defect. It is a major prognostic factor for survival in children with Down syndrome. Associate Ectopic ductal tissue (choice A) with coarctation of the aorta. Tetralogy of Fallot consists of a ventricular septal defect, an overriding aorta, pulmonic stenosis, and right ventricular hypertrophy. Itis the most common cause of early cyanosis and is caused by failure of the AP septum to align (choice B). Failure of neural crest cells to migrate (choice E) is endocardial cushion defect, common in Down syndrome.
-
Question 34 of 100
34. Question
1 pointsAll of the following are associated with Paget´s disease, EXCEPT
Correct
Incorrect
Explanation:
Paget´s syndrome is not associated with neurofibromatosis, but is associated with osteosarcoma, high output cardiac failure due to increased blood flow through affected bone, pathologic fractures, and osteoarthritis.
-
Question 35 of 100
35. Question
1 pointsCysteine proteinases are inhibited by which of the following?
Correct
Incorrect
Explanation:
α2 macroglobulin inhibits the cysteine proteinase family of enzymes, while the other molecules do not. Tissue inhibitor of metalloproteinases (TIMP) is a specific inhibitor for the metalloproteinase enzymes. Aprotinin inhibits mast cell tryptase. αl proteinase inhibitor inhibits elastase. αl chymotrypsin inhibits cathepsin G. -
Question 36 of 100
36. Question
1 pointsWhich of the following is NOT included in the HACEK group?
Correct
Incorrect
Explanation:
Eikenella corrodens is the other member of the HACEK group (the “E” in HACEK), which comprises a group of fastidious Gram- negative bacteria. Enterococcus is not part of the group. HACEK organisms can cause a subacute bacterial endocarditis. IV drug abusers and individuals with prosthetic valves are more at risk for HACEK infection.
-
Question 37 of 100
37. Question
1 pointsWhich of the following is NOT a metabolic alteration in severe acute hypoxia?
Correct
Incorrect
Explanation:
Lactic acid cannot be further metabolized, resulting in lactic acidosis. Normally in liver and muscle, metabolism of carbohydrates, the primary foodstuff, proceeds anaerobically up to the stage of pyruvate formation. Degradation of pyruvate requires oxygen. In hypoxic states, increasing proportions of pyruvate are degraded to lactic acid. Metabolic acidosis occurs, resulting in decreased total energy output from the breakdown of carbohydrates. The amount of energy available for continuing resynthesis of energy-rich phosphate compounds becomes inadequate.
-
Question 38 of 100
38. Question
1 pointsThe infectious agent that is the most common cause of uncomplicated urinary tract infection is
Correct
Incorrect
Explanation:
Escherichia coli is responsible for the majority of cases of uncomplicated urinary tract infections.
-
Question 39 of 100
39. Question
1 pointsAn 8-month-old boy is with symptoms of recurrent upper respiratory tract infections. He has mild facial hypoplasia and is hypocalcaemic. Microbiological investigations were normal and immunoglobulins were within normal limits. The infant´s immune function would show the following deficiency:
Correct
Incorrect
Explanation:
This child suffers from DiGeorge´s syndrome. Patients with DiGeorge´s syndrome often have near normal levels of immunoglobulins but with significant decreases in T cell numbers and relative increase in the percentage of B cells
-
Question 40 of 100
40. Question
1 pointsIn a diagnosis of Kawasaki´s disease, in which one of the following locations is vasculitis most often present?
Correct
Incorrect
Explanation:
Kawasaki´s disease, or mucocutaneous lymph node syndrome, most often involves the coronary vessels when vasculitis is present.
-
Question 41 of 100
41. Question
1 pointsThe most common cancer in American males is
Correct
Incorrect
Explanation:
Although lung cancer causes the most cancer deaths in American males, prostate cancer is the most common. The frequency of prostate cancer is followed by lung cancer (A) and colorectal cancer (C), respectively -
Question 42 of 100
42. Question
1 pointsA woman with history of multiple failed pregnancies and chronic pelvic pain undergoes a hysterectomy for palliation. Evaluation of the gross specimen reveals a 5 cm smoothly marginated submucosal rounded mass in the fundus, as well as multiple other smaller well-circumscribed subserosal and submucosal masses. Histological examination of the specimen would show
Correct
Incorrect
Explanation:
The lesions are the very common leiomyomas of the uterus which are well circumscribed, rubbers, white tan masses. Leiomyomas, if numerous or large, may cause chronic pelvic pain and infertility or pregnancy complications (if they intrude on the uterine cavity). Histologically, they are characterized by whorls of spindle-shaped smooth muscle cells. There is continued debate over whether there is malignant potential associated with fibroids or whether leiomyosarcomas arise de novo. Heterogeneous cells with components of endoderm and ectoderm describes a teratoma, which is unlikely to be multiple or intrauterine. Increased number of columnar cells with preservation of the basement membrane describes endometrial hyperplasia, which would not present with subserosal masses. Moderately differentiated cells with a striated pattern describe rhabdomyomas, which are benign tumors of skeletal muscle. Poorly differentiated cells with abundant atypical mitotic figures describe malignant neoplasm, which would not be so well defined.
-
Question 43 of 100
43. Question
1 pointsWhat is the most common genetic disorder of the pancreas?
Correct
Incorrect
Explanation:
Cystic fibrosis is the most common genetic disorder of the pancreas (approximately 1 in 2,000 births). The other genetic disorders are less common. Shwachman-Bodian syndrome is the combination of pancreatic insufficiency and bone marrow dysfunction.
-
Question 44 of 100
44. Question
1 pointsThe principal type of mercury exposure in the general population is which of the following?
Correct
Incorrect
Explanation:
The most common type of mercury exposure in the general population is dietary. The major source is fish. The other sources are less common.
-
Question 45 of 100
45. Question
1 pointsThe most commonly mutated gene in the progression of human cancer is which one of the following?
Correct
Incorrect
Explanation:
p53, a tumor suppressor gene, is found to be mutated in over 50% of all types of human cancers.
-
Question 46 of 100
46. Question
1 pointsWhich of the following is NOT a complication of chronic pancreatitis?
Correct
Incorrect
Explanation:
Cobalamin (Vitamin B12 deficiency) probably results from excessive binding of cobalamin by nonintrinsic factor cobalamin binding proteins, which are normally destroyed by pancreatic proteases. Icterus may result from either edema of the head of the pancreas, with compression of the common bile duct, or from chronic cholestasis secondary to chronic inflammation around the intrapancreatic portion of the common bile duct. Although gastrointestinal bleeding may result from pathology of the gastrointestinal tract itself, it may also occur if a pseudocyst erodes into the duodenum. Varices, resulting from splenic vein thrombosis due to inflammation of the tail of the pancreas, may also rupture. Bone pain may occur due to intramedullary fat necrosis. Subcutaneous fat necrosis may present as tender red nodules on the lower extremities.
-
Question 47 of 100
47. Question
1 pointsAn 18 year old mentally retarded boy is brought because of recurrent epilepsy which is uncontrolled. Exam shows facial nevi and nodules in the retina. No other visible rashes on the body are present. His sister has similar symptoms. The most likely diagnosis is
Correct
Incorrect
Explanation:
Tuberous sclerosis is an autosomal dominant neurocutaneous disorder which presents with seizures and adenoma sebaceum. Syringomyelia is a neurological disorder presenting without a rash and is secondary to a cavity within the central spinal cord. Neurofibromatosis is an autosomal dominant disorder. Café-au-lait spots are seen which are macular and greater than 1.5 cm in diameter, presenting mainly on the trunk and axilla and greater than six in number. Arnold Chiari malformation is secondary to a developmental displacement of the cerebrellar tonsils through the foramen magnum. Progressive cerebellar ataxia occurs. There is no rash. Meningococcal meningitis can be rapidly fatal if not treated early. The patient would appear toxic and have a rash on the lower extremities or trunk.
-
Question 48 of 100
48. Question
1 pointsWhich one of the following is associated with gastric carcinoma metastases to an umbilical node?
Correct
Incorrect
Explanation:
Gastric carcinoma may metastasize to a number of places, including the left supraclavicular lymph node (Virchow´s node), an umbilical lymph node (Sister Mary Joseph´s sign), the rectal shelf (Blumer´s shelf), and the ovary (Krukenberg´s tumor). A Klatskin´s tumor is a pancreatic adenocarcinoma that arises at the confluence of the hepatic ducts. -
Question 49 of 100
49. Question
1 pointsAn 18 year old boy is found to have his cardiac shadows reversed on a routine chest X-ray. He also has bouts of frequent sinusitis and respiratory tract infections. He is ppd positive. The most likely diagnosis is which one of the following?
Correct
Incorrect
Explanation:
This patient has Kartagener´s syndrome, which is caused by immotile cilia from a defect in the protein dynein. Situs inversus, sinusitis, bronchiectasis, and infertility are seen. The cilia in the respiratory tract and sperm are immotile and microfilament movement is impaired. Pulmonary tuberculosis would show upper lobe infiltrates or effusions. It is very common to have a positive ppd in endemic areas. Kallmann´s syndrome is mainly seen in males and is associated with hypogonadotropic hypogonadism. It is familial and associated with anosmia and mental retardation. Wegener´s granulomatosis is a necrotizing vasculitis associated with renal failure, upper respiratory problems, and lung nodules. Turner´s syndrome is associated with a female phenotype. Patients have short stature, webbed necks, and streaked gonads. The genotype is (45, X).
-
Question 50 of 100
50. Question
1 pointsAll of the following cytokines are identified in rheumatoid synovial fluid, EXCEPT
Correct
Incorrect
Explanation:
Lymphokine activated killer cells are one of several cellular defenses exploited in cancer therapy. The local production of cytokines probably accounts for many of the pathological and clinical findings in rheumatoid arthritis. Tumor necrosis factor alpha and interleukin-8 are chemotactic for neutrophils. They, as well as granulocyte macrophage colony-stimulating factor, activate polymorphonuclear leukocytes. Interleukin-1 activates chondrocytes, fibroblasts, osteoblasts, and chondroblasts, resulting in bone resorption.
-
Question 51 of 100
51. Question
1 pointsKernicterus is developed by deposition which compoung in the basal ganglia?
Correct
Incorrect
Explanation:
Accumulation of unconjugated bilirubin in the basal ganglia is responsible for this syndrome of bilirubin encephalopathy. Iron, conjugated bilirubin, biliverdin, and galactosamine are not involved in kernicterus.
-
Question 52 of 100
52. Question
1 pointsA patient has intermittent, severe pain in the right upper quadrant of the abdomen, especially after fatty meals. Ultrasound shows multiple small opacities in the gallbladder. A risk factor this patient´s condition is
Correct
Incorrect
Explanation:
The patient has gallstones (cholelithiasis). A useful mnemonic to remember the risk factors for gallstones is the 5 F´s: female, fat, fertile, forty, and flatulent. Gallstones are usually composed predominantly of cholesterol with lesser amounts of other components such as calcium salts, bile acids, and bile pigments. Exceptions to this rule are nearly pure cholesterol stones and pigment (calcium bilirubinate) stones. The incidence of mixed stones and cholesterol stones is also increased in association with Crohn disease, cystic fibrosis, clofibrate therapy, estrogen therapy, rapid weight loss, and Native American origin. Risk factors for pigment stones include chronic hemolysis, alcoholic cirrhosis, and biliary infection. The typical age for gallstones is 40 years or older, not 20 years. Female, not male sex predisposes the individual for gallstones. Many patients with gallstones are multiparous (fertile), not nulliparous. Gallstones are associated with obesity (fat), not a thin build
-
Question 53 of 100
53. Question
1 pointsA germline B lymphocyte is determined to possess 200 V gene segments, 20 D gene segments, and 50 J gene segments to encode heavy-chain variable domains. It also possesses 100 V gene segments and 10 J gene segments to encode light-chain variable domains. How many distinct idiotypes can be produced in the absence of recombinational inaccuracies?
Correct
Incorrect
Explanation:
During their development, progenitor B and T lymphocytes undergo complex gene segment rearrangements that allow them to create a wide diversity of antigen recognition molecules. In the case of a B lymphocyte, three gene segments (V, D, and J) are randomly recombined to create the coding for the variable domain of the heavy chain of immunoglobulin. Two gene segments (V and J) are randomly recombined to create the coding for the variable domain of the light chain of immunoglobulin. Analogous rearrangements are made in T cells to create the variability of the N-terminal domains of the T-cell receptor beta or alpha chains (respectively). In order to determine the maximum number of distinct idiotypes (antigen combining sites) in this question, you simply multiply the different possible choices: for the heavy chain, 200 x20 x50, or 2 x 10^5 possibilities; for the light chain, 100 x 10, or 1 x 10^3 possibilities. Since heavy and light chains randomly associate following rearrangement, the product of the two possibilities (2 x10^5) x (1 x10^3), or 2 x 10^8 gives the total number of possible combinations. Option A, 1,000 is not correct. This is the number of distinct light-chain variable domains that could be produced. Option B 4000, is not correct. This is the product of they and D gene segment rearrangements only for the heavy chain. Option C, 1 x 10^4, is not correct. There is no way to arrive at this figure by multiplying combinations of VDJ gene segments. Option D, 2 x 10^5, is not correct. This is the total number of heavy-chain variable domains that could be produced, but an immunoglobulin idiotype consists of both heavy and light-chain variable domains.
-
Question 54 of 100
54. Question
1 pointsA 72 year old nursing home resident develops fever of 39oC (102.2oF) and shortness of breath. He has a productive cough of brown-streaked sputum filled with gram positive diplococci. CXR reveals consolidation in the lower left lobe of the lung. Immune cells that will be most numerous in the pathologic lesions in this man´s lungs include Correct
Incorrect
Explanation:
This man has the symptoms of a “typical” pneumonia caused by Streptococcus pneumoniae. Typical pneumonia is characterized by high fever, lobar consolidation, and productive sputum. These symptoms are a result of the primary responding immune cell, the neutrophil, which enters the area, releases pyrogenic products (fever), causes innocent bystander damage to the lung (sputum), and creates abscesses (lobar consolidation). Most agents causing typical pneumonia are extracellular organisms, which possess molecules that are chemotactic for neutrophils (f-Met peptides, peptidoglycan, teichoic acids). B lymphocytes are most frequently found in the secondary lymphoid organs (lymph nodes and spleen). They are not primarily associated with the pathology of typical pneumonia. Eosinophils predominate in lesions that are either parasitic or allergic in nature. Since neither of these cases is true here, they would not be expected to predominate in these lesions. Macrophages are called into areas of infection by the cytokines produced by T lymphocytes. Macrophages and T lymphocytes are responsible for the pathology in atypical pneumonias but not in the pathology of typical pneumonia. T lymphocytes are stimulated by presentation of antigens on the surface of antigen- presenting cells. They are thus primarily stimulated by intracellular pathogens, whose antigens are presented in the groove of the MHC class II molecule of various phagocytic cells. This reaction is responsible for the production of alveolar septa swollen with mononuclear cells, which is the primary manifestation of an atypical pneumonia.
-
Question 55 of 100
55. Question
1 pointsThe type of breast cancer that has the best prognosis is which one of the following?
Correct
Incorrect
Explanation:
Breast adenocarcinoma occurs in a variety of morphologic subtypes some of which have a significantly worse or better prognosis than the most common invasive ductal carcinoma. Tubular carcinoma occurs in women younger than 50 and has an excellent prognosis. It consists of well formed tubules which are so well differentiated that the tumor is sometimes mistaken for a benign lesion. Axillary metastases are present in fewer than 10% of cases at the time of diagnosis. Of the histologic types of breast cancer listed here, the tubular variant is certainly associated with the best chances of survival. Invasive ductal carcinoma and invasive lobular carcinoma are the main morphologic types of breast carcinoma. Together, they constitute 80 to 90% of all cases. The prognosis mainly depends on the staging related to size of the primary cancer and evidence of lymphohematogenous spread to regional nodes and distant organs. Grading (that is, degree of differentiation of primary tumor) has a minor influence on prognosis.
Whenever breast cancer of any histologic type grows very rapidly, infiltrating and occluding the dermal lymphatics, acute swelling and redness of the breast may develop, leading to a clinical picture referred to as inflammatory carcinoma. Inflammatory carcinoma, therefore, is not a true morphologic subtype but a clinical definition. Any form of breast cancer manifesting as “inflammatory carcinoma” is likely to be extremely aggressive. Medullary carcinoma has a slightly better prognosis than ductal or lobular types. It tends to occur in younger women, especially those with mutations of BRCA1 Grossly, this tumor is soft and well circumscribed. Histologically, this carcinoma is surrounded by a marked lymphoplasmacytic reaction, which may account for its propensity to grow more slowly than other forms of cancer. Medullary carcinoma, however, has a worse prognosis than tubular carcinoma. -
Question 56 of 100
56. Question
1 pointsA 44 year old male with recurrent headaches also has impotence and loss of libido that has gradually worsened during the past year. A bitemporal hemianopia is also present. Serum prolactin levels are increased while serum LH and testosterone are decreased. The most likely diagnosis is
Correct
Incorrect
Explanation:
Hyperprolactinemia the most common hypothalamic-pituitary disorder. A tumor in the pituitary (prolactinoma) that secretes excessive prolactin is the most common functional pituitary tumor. The increase in serum prolactin suppresses the normal GnRH-gonadotropin gonadal steroid axis. Hypogonadism, manifested as amenorrhea in females or loss of libido and/or impotence in males, is a prominent symptom. Blood levels of sex steroids are usually decreased. Although not present in this patient, galactorrhea may occur due to the action of prolactin on the mammary gland. Since the anterior pituitary is located just below the optic chiasm, space filling tumors that compress this structure may produce visual field defects. Craniopharyngioma can also produce hyperprolactinemia. If this tumor impinges on the pituitary stalk and interferes with hypophyseal portal blood flow, the decrease in delivery of dopamine to the anterior pituitary can result in increased prolactin secretion. However, craniopharyngioma is more common in children and adolescents than in adults. Only 20% of craniopharyngiomas are diagnosed after age 40. Idiopathic panhypopituitarism would manifest with decreases in all anterior pituitary hormones, including prolactin. Isolated LH deficiency could explain the loss of libido and decreased plasma levels of LH and testosterone. However, it could not explain the increase in prolactin. Pituitary infarction, which can occur in women who hemorrhage excessively during parturition (Sheehan syndrome), leads to varying degrees of hypopituitarism. If the infarction produced significant necrosis in pituitary lactotrophs, blood levels of prolactin would be low rather than high.
-
Question 57 of 100
57. Question
1 pointsConjoined twins are born attached at the chest. Examination of the placenta would likely reveal what arrangements of the fetal membranes
Correct
Incorrect
Explanation:
This question may seem a bit challenging, but you have all the information you need to answer it. The type of fetal membranes produced in identical twin pregnancies depends on the timing of the twinning process. The chorion forms before the amnion, so the possible combinations for all twin pregnancies are monoamnionic and monochorionic, diamnionic and monochorionic, and diamnionic and dichorionic (with the chorions either separate or fused). The first two possibilities are seen only in identical twins, the last one can be seen in both identical and fraternal twins. Exactly what happens depends on the precise point at which twinning occurs. Very early separation (at the two cell stage) produces completely separate membranes with duplication of both the amnion and chorion. Slightly later separation results in two amnions and fused chorions. Separation at a later stage, when the inner cell mass is present, produces twins with one chorion and two amnions. Very late separation results in one chorion and one amnion. Conjoined (Siamese) twins result from a very late twinning event, so the placenta will be monoamnionic and monochorionic. Monoamnionic and dichorionic placentas are not usually seen, as the chorion forms before the amnion.
-
Question 58 of 100
58. Question
1 pointsAn accident in a dry cleaning facility exposes a worker to massive amounts of carbon tetrachloride, both on the skin and by inhalation. Which of the following organs is most likely to suffer severe damage?
Correct
Incorrect
Explanation:
The most important risk to this patient is liver damage by free radical species derived from carbon tetrachloride (CCl4), notably CCl3. These species are produced when the hepatic P450 microsomal system attempts to degrade the CCl4. They can cause severe, sometimes fatal, fatty liver damage by reacting with the polyenoic (multiple double bonds) acids present in the membrane phospholipids. The reaction is particularly harmful because peroxides are often formed as a byproduct. Peroxides can be auto-catalytic, in that the peroxide radicals themselves form new free radicals capable of more damage to membranes and other cellular structures. Clinically, patients may have an extremely rapid (30 min to 2 hours) decline in hepatic function after the carbon tetrachloride exposure. Other organs are not affected to the same degree, because of the relatively lower concentration of the toxic metabolites there
-
Question 59 of 100
59. Question
1 pointsA female presents with bilateral discharge of fluid from her nipples is suspected to have a hormone secreting pituitary tumor. Which hormone closest structural homology to the hormone likely responsible for her nipple discharge?
Correct
Incorrect
Explanation:
The hormone secreted by the pituitary that would cause nipple discharge (galactorrhea) is prolactin. Prolactin and growth hormone have very close structural homologies, and are considered to form the “prolactin-growth hormone family.” ACTH and melanocyte stimulating hormone (MSH) are in the “ACTH family.” FSH, LH, and TSH, have similarities, and form the “glycoprotein hormone family.”
-
Question 60 of 100
60. Question
1 pointsA 15 year old boy is brought in a very confused state. He is very thirsty and consumes a large amount of water while waiting for service. An alert resident notes that his breath smells like acetone. His presentation would be most consistent with which change in electrolyte status?
Correct
Incorrect
Explanation:
This patient has diabetic ketoacidosis (DKA), as noted by the acetone smell on the breath. Metabolic acidosis tends to cause a shift of potassium from the intracellular compartment to the extracellular fluid. This increases plasma potassium concentration and, along with other effects of chronic acidosis, causes increased urinary potassium excretion. The result is progressive depletion of potassium from the body; patients in severe DKA have been found to have as much as 60% reduction of total body potassium while still displaying hyperkalemia. Volume depletion caused by diuresis can potentially cause hypernatremia however glycosuria increases sodium excretion. The combination of natriuresis with large volume consumption of water strongly suggests that hyponatremia will be present. Hyperphosphatemia is incorrect, because diabetic ketoacidosis
causes excessive loss of phosphate in the urine, and therefore hypophosphatemia. Hypokalemia is incorrect, because, as noted above, metabolic acidosis causes a shift of potassium from the intracellular fluid into the extracellular fluid; hyperkalemia is expected. Positive potassium balance is in correct in the setting of chronic ketoacidosis (duration greater than about 24 hours). The patient´s condition is consistent with DKA of several days duration. It is important to remember that although acute metabolic acidosis decreases urinary potassium excretion, sustained acidosis has the opposite effect. -
Question 61 of 100
61. Question
1 pointsA patient is brought into the emergency department with lethargy and confusion. He is type 2 diabetic and is currently taking metformin. He also admits to frequent consumption of alcohol. Laboratory values show:
- PaO2 = 90mmHg
- PaCO2=19mmHg
- pH=7.30
Assuming a respiratory exchange ratio of 1, which pulmonary condition is most likely to be present?
Correct
Incorrect
Explanation:
The alveolar PO2 is found from the alveolar gas equation:
PAO2 = PIO2 – PACO2 /R
Plo2 = .21 (P barometric – P water vapor)
P barometric = 760 mm Hg
water vapor = 47
then P102 = (.21)(713) = 149mm Hg. Therefore PAQ2 = 149-(19/1) = 130mm Hg. Thus, the alveolar-arterial difference is 130-90 =40 suggesting some underlying lung disease that is impairing oxygen uptake in the lungs.
Option A is incorrect because the decreased arterial PCO2 indicates increased alveolar ventilation. Option B is incorrect because only severe hypoxia (PO2 < 50 mm Hg) would cause alveolar ventilation to double, as it has in this patient. Primary respiratory acidosis would display increased, rather than decreased, arterial PO2. Acidosis shifts the O2 dissociation curve to the right, increasing the P50. -
Question 62 of 100
62. Question
1 pointsA 41 year old male has increasing difficulty over the past 3 years. He has a feeling of pressure in his chest occurring 2-3 seconds after swallowing a solid bolus. He also regurgitates undigested food eaten hours before. Barium swallow shows distended esophageal body with a smooth tapering at the swallowing and a lower esophageal sphincter that fails to relax. The likely diagnosis is
Correct
Incorrect
Explanation:
Achalasia is an acquired esophageal motility disorder that slowly develops. The motility is abnormal due to the loss of inhibitory enteric neurons of the esophageal body and lower esophageal sphincter (LES). Both vasoactive intestinal peptide and nitric oxide function as inhibitory neurotransmitters here, and the presence of both is decreased in achalasia. Radiographs typically show a dilated esophagus that tapers at the lower esophageal sphincter, producing a so called “bird´s beak.” Because of the poor motility, ingested food is regurgitated and can lead to aspiration symptoms. Heartburn can occur due to production of lactic acid in the esophagus as the retained food is fermented. Manometric demonstration of absent peristalsis in the esophageal body, and poor relaxation of the lower esophageal sphincter with a swallow confirm the diagnosis. The vast majority of cases of achalasia are of unknown etiology. A very small number arise as complications of other entities such as Chagas disease. However, in order to consider this diagnosis, a history of recent travel to an endemic area. (South America) is very important. The primary complaint with diffuse esophageal spasm is mid-sternal pain that can be misdiagnosed as cardiac pain. The pain is caused by prolonged contraction of the entire esophageal body. Symptoms can be brought on by eating certain hot or cold meals. A manometric study may show poor peristalsis in the smooth muscle portion of the esophageal body, but lower esophageal sphincter function is unaffected. The primary complaint with incompetent lower esophageal sphincter is heart burn and regurgitation due to gastroesophageal reflux. Endoscopic examination of the esophagus may reveal inflammation, erosions, and even ulcers. A manometric study would show lower than normal resting tone in the lower esophageal sphincter, or a sphincter that relaxes inappropriately.
Scleroderma is a connective tissue disease in which esophageal smooth muscle is gradually replaced by dense collagenous material. Manometry would show poor esophageal peristalsis and decreased lower esophageal sphincter tone. Significant acid reflux with resultant esophagitis is almost universal. -
Question 63 of 100
63. Question
1 pointsA hospitalized patient with hemoglobinuria and symptoms of anemia undergoes evaluation. His reticulocyte count is elevated. Remaining lab results are pending. Which diagnosis is most consistent with the information available so far?
Correct
Incorrect
Explanation:
Hemoglobinuria means that there is hemoglobin in the urine and suggests hemolysis in a patient with symptoms of anemia. The patient´s reticulocytosis further supports this suspicion. Reticulocytosis refers to an increase in the number of circulating reticulocytes. Reticulocytes are immature red blood cells that typically constitute approximately 1% of circulating red blood cells. The reticulocyte count is used to differentiate between anemias due to decreased erythropoiesis (low reticulocyte count since marrow can´t produce reticulocytes) and anemias secondary to blood loss or peripheral red blood cell destruction (hemolysis; reticulocyte count elevates as marrow tries to compensate). Of the choices listed, only G6PD deficiency leads to hemolysis. (In the absence of G6PD, there is a deficiency of NADPH and, therefore, a deficiency of reduced glutathione. This causes oxidative stress on the erythrocyte that leads to hemolysis.) Other causes of hemolytic anemia include hereditary spherocytosis, sickle cell disease, thalassemias, autoimmune hemolytic anemia, red blood cell trauma, and paroxysmal nocturnal hemoglobinuria. All the distracters would lead to anemia secondary to decreased erythropoiesis and would therefore be associated with low reticulocyte counts. Chronic renal disease results in decreased production of the hormone erythropoietin, leading to decreased reticulocyte (and therefore erythrocyte) production. Folate deficiency results in decreased red blood cell production and a megaloblastic anemia characterized by macrocytic red blood cells and hyper-segmented neutrophils. Iron deficiency results in a microcytic, hypochromic anemia. An elevated reticulocyte count after administration of ferrous sulfate would indicate improvement of the anemia with therapy. Pernicious anemia refers to vitamin B12 malabsorption secondary to antibodies against intrinsic factor or intrinsic factor receptors. B12 deficiency leads to megaloblastic anemia.
-
Question 64 of 100
64. Question
1 pointsA 33 year old man was revived after having been submerged underwater for 15 minutes. The physicians were concerned that he sustained hypoxic damage to his brain. The structure that would most likely be damaged is
Correct
Incorrect
Explanation:
The Purkinje cell layer of the cerebellar cortex, pyramidal cells in the CA1 subfield of the hippocampus, and pyramidal cells in layers 3 and 5 of the cerebral cortex are all very sensitive to the effects of cerebral hypoxia and ischemia. The basis for this selective vulnerability may be NMDA-receptor mediated excitotoxic injury of these specific populations of neurons. The claustrum is a thin sheet of gray matter located between the external capsule and the extreme capsule. It is not selectively targeted by hypoxic/ischemic injury to the brain.
Layer 2 of the cerebral cortex is the external granular layer, populated by closely packed granule cells. This layer is not affected by ischemia to the same extent as layers 3 and 5, the external and internal pyramidal cell layers. The mammillary bodies are hypothalamic structures located at the base of the brain. They are damaged in Wernicke Korsakoff syndrome, which is due to thiamine deficiency. The subiculum lies medial and inferior to the hippocampus. The subiculum is not selectively vulnerable to hypoxia, in contrast to the nearby CA1 subfield of the hippocampus. -
Question 65 of 100
65. Question
1 pointsA girl aged 18 years whose grandmother was recently diagnosed with breast cancer discovers a large, round, moveable nodule in her left breast. She visits her physician. Which diagnosis should rank highest on the physician´s differential?
Correct
Incorrect
Explanation:
Fibroadenoma is the most common benign breast tumor. It occurs in women of reproductive age, generally before age 30, and may be related to increased estrogen sensitivity. It presents as a single, movable breast nodule, not fixed to the skin. Surgical excision is required for definitive diagnosis.
Cystosarcoma phyllodes is a fibroadenoma like tumor that has become large, cystic, and lobulated. It may contain malignant elements. Fibrocystic breast disease is the most common breast disorder. It usually affects women older than 35. It involves a distortion of the normal breast changes associated with the menstrual cycle. Patients often have lumpy, tender breasts, especially during the several days prior to menstruation. Infiltrating ductal carcinoma is the most common type of breast cancer. It occurs most frequently after age 40. Other risk factors include nulliparity, family history, early menarche, late menopause, previous history of breast cancer, obesity, and high fat diet. Intraductal papilloma is associated with bloody or serous nipple discharge. It is most common in women aged 20-50. -
Question 66 of 100
66. Question
1 pointsA 25 year old man with a long history of heavy alcohol intake is admitted with nausea and frequent vomiting 4 hours after a meal. During review he vomits a cupful of blood. The cause of his haematemesis is Correct
Incorrect
Explanation:
Persistent vomiting can eventually lead to small tears in the oesophegus leading to the vomiting of red blood. Varices would produce large volumes of blood (much more than just cupful.
-
Question 67 of 100
67. Question
1 pointsTrue statement regarding IgE is which one of the following?
Correct
Incorrect
Explanation:
IgG is the predominant form of immunoglobulin in plasma at a concentration around 10,000 times that of IgE. IgG crosses the placenta to confer immunity to the fetus but IgE does not. IgE is involved in arming mast cells and basophils. IgE causes mast cells to release vasoactive amines, such as histamine, producing an inflammatory response which can result in a type I hypersensitivity reaction. IgE is responsible for allergen mediated diseases such as anaphylaxis, asthma and atopy.
Total serum IgE is frequently increased in those with atopy but serum IgE does not rise acutely during an asthmatic attack. -
Question 68 of 100
68. Question
1 pointsA 47 year old woman develops nephritic syndrome and is awaiting further tests to establish the underlying aetiology. Corticosteroids would be most effective in reversing the nephritic syndrome in which condition?
Correct
Incorrect
Explanation:
Although there is no known effective treatment for IgA nephropathy, there have been reports of favourable response to long term corticosteroid therapy. 80% adults with minimal change glomerulonephritis (GN) will respond to steroids, although remissions can take up to 16 weeks. Membranous GN does not respond to steroid treatment. No specific treatment is available to cause regression of amyloid deposits.
-
Question 69 of 100
69. Question
1 pointsMutation in which of the following is associated with Ehler-Danlos syndrome?
Correct
Incorrect
Explanation:
The mutations in the COL1A1 gene that cause the arthrochalasia type o Ehlers-Danlos syndrome lead to a pro-alpha1 (I) chain that is missing critical segment. The absence of this segment interferes with the assembly and structure of type I collagen molecules and their processing into collagen fibrils. Tissues that are rich in type I collagen, such as the skin, bones, and tendons, are affected by this change, which leads to the characteristic features of this type of Ehlers-Danlos syndrome
-
Question 70 of 100
70. Question
1 pointsA 41-year-old male is diagnosed with dystrophia myotonica.Which one of the following features would be expected in this patient?
Correct
Incorrect
Explanation:
Dystrophia myotonica is an autosomal dominant condition with variable penetrance. Symptoms characteristically begin from the age of 20-30 with weakness and myotonia.
- Ptosis
- Cataracts
- Frontal baldness
- Gynaecomastia
- Diabetes and
- Reduced reflexes with myotonia are features.
- Progressive external ophthalmoplegia is a feature of ocular muscular dystrophy.
-
Question 71 of 100
71. Question
1 pointsA 26-year-old man presents with proteinuria, haematuria and sensorineural deafness. Which of the following protein structures is likely to be abnormal?
Correct
Incorrect
Explanation:
The diagnosis is Alport´s syndrome, which is a disorder of Type 4 collagen assembly and is inherited as an X-linked (most likely to recessive) disorder in 85% of cases. Fibrillin gene abnormalities are associated with Marfan´s syndrome. Type 1 collagen disorders are associated with osteogenesisimperfecta; it is the main type of collagen in tendon and bone. Type III Collagen is the main component of reticular fibres.
-
Question 72 of 100
72. Question
1 pointsWhich one of the following is highly specific for Wegner´s granulomatosis?
Correct
Incorrect
Explanation:
When requesting an ANCA test, both immunofluorescence and an ELISA test are generally performed. On immunofluoresecence, if ANCA are present the staining pattern; the staining pattern may be cytoplasmic (cANCA) or perinuclear (pANCA). Typical antigen specificity includes proteinase 3 or myeloperoxidase. cANCA and specificity for the PR-3 antigen is most specific for Wegener´s granulomatosis. This pattern is also seen in microscopic polyarteritis nodosa and rarely Churg-Strauss syndrome. PANCA and MPO are less specific findings detected in various vasculitic illnesses and occasionally in chronic infections.
-
Question 73 of 100
73. Question
1 pointsWhich of the following is NOT associated with intrauterine growth retardation?
Correct
Incorrect
Explanation:
Maternal diabetes mellitus is associated with large for date babies, not with intrauterine growth retardation. Autosomal trisomies, cytomegalovirus infection, toxemia, and maternal cigarette smoking are all causes of intrauterine growth retardation.
-
Question 74 of 100
74. Question
1 pointsThe most common type of chronic arthritis in adults is
Correct
Incorrect
Explanation:
Although rheumatoid arthritis is the most common of all the autoimmune disorders, it is osteoarthritis that is the most common of all forms of arthritis in adults.
-
Question 75 of 100
75. Question
1 pointsWhat is the most common lesion responsible for diabetic nephropathy?
Correct
Incorrect
Explanation:
Interstitial nephritis can cause diabetic nephropathy but is much less common than diffuse glomerulosclerosis. IgA deposition and glomerulonephritis are unusual in diabetes. Nephrolithiasis may be associated with diabetic nephropathy but is much less common than.
-
Question 76 of 100
76. Question
1 pointsWhich of the following is NOT required for paroxysmal supraventricular tachycardia to occur?
Correct
Incorrect
Explanation:
Reentry arrhythmias, such as supraventricular tachycardias, are disorders of impulse propagation. They are self sustained, but not self-initiated. Thus, an initiating premature complex is required. For reentry to begin, a one way conduction block must be present. A circuit is formed by either an anatomic or functional barrier. Since wavelength is dependent on the conduction velocity and the refractory period the conduction velocity must be very slow and the refractory period very short (or both) for reentry to occur.
-
Question 77 of 100
77. Question
1 pointsWhich of the following condition is NOT associated with hypoglycemia?
Correct
Incorrect
Explanation:
Glucagonomas release glucagon and, thus, elevate serum glucose. Insulinomas secrete insulin, which lowers serum glucose. Alcoholism can also lower the serum glucose, at least in part, because of depletion of hepatic glycogen. Adrenal insufficiency may be associated with decreased glucose concentrations, as may pituitary insufficiency.
-
Question 78 of 100
78. Question
1 points“Scleroderma” is best matched by which one of the following answer?
Correct
Incorrect
Explanation:
Scleroderma is associated with calcinosis.
-
Question 79 of 100
79. Question
1 pointsThe bone marrow finding that is diagnostic of Hodgkins´s disease is which one of the following?
Correct
Incorrect
Explanation:
Reed Sternberg cells found in the bone marrow are large, bilobed cells with prominent eosinophilic nucleoli, thick nuclear membrane, and relatively abundant cytoplasm. A stromal background is required for the Reed Stemberg cell in order to make the histological diagnosis of Hodgkin´s disease. Abundance of plasma cells in the bone marrow usually make the diagnosis of multiple myeloma. Multinucleated giant cells seen on the Tzanck prep make the diagnosis of herpes. Excess megakaryocytes seen in the bone marrow produce excess platelets, and thus the diagnosis of essential thrombocytosis is made. Blast cells are seen in leukemias and imply a poor prognosis.
-
Question 80 of 100
80. Question
1 pointsA 37 year old man presents with severe unilateral flank pain that radiates to the groin. He has a normal serum calcium level and hematuria. Urine pH is below 7.0. The patient is afebrile. No visible opaque stones are present on the IV pyelogram. The most likely diagnosis isaA 37 year old man presents with severe unilateral flank pain that radiates to the groin. He has a normal serum calcium level and hematuria. Urine pH is below 7.0. The patient is afebrile. No visible opaque stones are present on the IV pyelogram. The most likely diagnosis is
Correct
Incorrect
Explanation:
Uric acid stones in the kidneys are the only ones which are radiolucent. They account for 5% of all stones and are secondary to excessive purine intake. The urine pH is usually very low in urate nephrolithasis. Calcium oxalate/carbonate nephrolithiasis is the most common form of kidney stones and accounts for 75% of all kidney stones. These stones are secondary to hypercalcemia and are usually radio opaque. Cystine stones contain sulfur. They are caused by a hereditary defect of amino acid transport in the renal proximal tubules. Struvite stones are secondary to chronic urinary tract infections with urea splitting organisms such as proteus. These stones are composed of ammonium, magnesium, phosphate, and calcium. They sometimes form staghorn calculi which have to be removed surgically. Oxalate stones are rare less than 5% of all kidney stones. They are secondary to ileal resection or bypass.
-
Question 81 of 100
81. Question
1 pointsWhich one of the following is the most common underlying condition in patients with an episode of acute pancreatitis?
Correct
Incorrect
Explanation:
Alcoholism.
-
Question 82 of 100
82. Question
1 pointsA 6 months girl who was normal at birth begins to show signs of motor retardation. Although she could sit up at 5 months she can no longer do so. She continues to deteriorate and eventually becomes unresponsive to visual or auditory stimuli. Funduscopic examination reveals a cherry red spot. The genetic abnormality that is most often related to the development of this disease is Correct
Incorrect
Explanation:
Most cases of Tay Sachs produces a frameshift mutation, resulting in the creation of a premature stop codon downstream from the inserted nucleotides. The disease presents as described in the question and is more common in Ashkenazi Jews. The carrier rate in this population is 1 in 25. There is currently no effective treatment, and survival beyond 4 years of age is rare. Confined placental mosaicism is caused by a genetic aberration (e.g., a trisomy), occurring in the trophoblast or extraembryonic progenitor cells in the inner cell mass of the embryo. This leads to abnormal placental development and intrauterine growth retardation. However, no embryologic mutation or abnormality is present. Expansion of tandem nucleotide repeats is thought to be etiologically involved in fragile X syndrome and Huntington disease. The latter is a devastating neurologic disease that typically affects individuals in the 4th and 5th decades of life, prior to which they are completely normal. The former is an X-linked disorder and is thus much more common in males. The neurologic deterioration is not as dramatic as in Tay Sachs. Fragile X patients reach early developmental milestones, after which they experience declining 10 with increasing age, autistic like behavior, and aggressive tendencies. Motor deficits are not prominent in this disease. Non-disjunction during meiosis is a common cause of trisomy, e.g., trisomy 21, or Down syndrome. Down syndrome children have very characteristic features, including up-slanting palpebral fissures, bilateral epicanthal folds, a hypoplastic nasal bone, and a flat nasal bridge. They are generally cheerful in affect. There is no progressive decline in these children; the mental retardation is consistent from birth. Translocations are frequently involved in leukemias, lymphomas, and solid tumors. A minority of cases of Down syndrome are actually due to a translocation rather than to trisomy. -
Question 83 of 100
83. Question
1 pointsA 46 year old Boy Scout notices flu like symptoms two weeks following a camping expedition. He presents after a low grade fever, myalgia, and malaise have been present for several days. Examination reveals an erythematous rash on one calf that consists of concentric rings. The agent that is transmitted by the same vector as the infectious agent in this condition is which one of the following?
Correct
Incorrect
Explanation:
This man has the symptoms of Lyme disease, caused by the large spirochete, Borrelia burgdorferi. The vector of Lyme disease is the Ixodes tick, which is also the vector which transmits Babesia microti, a sporozoan parasite related to Plasmodium, and Ehrlichia, which is an organism related to the Rickettsiae. The rickettsiae are prokaryotes with a cell wall, so they are not represented on the list.
A eukaryote with a cell wall describes a fungus; yet there are no fungi transmitted by the Ixodes tick. An infectious protein is a prion. These are the agents of so called slow viral diseases; and they are not known to be arthropod transmitted.
A prokaryote without a cell wall would describe the family Mycoplasmataceae, which include the genera Mycoplasma and Ureaplasma. A prokaryote without muramic acid in its cell wall describes the Chlamydiaceae, which include the genera Chlamydia and Chlamydophila -
Question 84 of 100
84. Question
1 pointsA woman with metastatic breast adenocarcinoma has an intravenous catheter placed for the delivery of her weekly chemotherapeutic drugs. 2 weeks after her first treatment, she develops fever. Blood culture reveals gram positive, non hemolytic cocci growing in clusters. What additional feature is likely to define the genus and species of the infectious agent
Correct
Incorrect
Explanation:
The finding of gram positive cocci in clusters describes the genus Staphylococcus. The fact that the staphylococci cultured are non hemolytic rules out S. aureus and leaves S. epidermidis as the most likely causative agent. This is a common cause of subacute bacterial endocarditis, particularly following infection of intravenous catheters or shunts. S. epidermidis is distinguished by all other staphylococci in that it is coagulase negative, non hemolytic, and novobiocin sensitive. Bacitracin resistance distinguishes Streptococcus agalactiae from S. pyogenes (which is bacitracin sensitive). Catalase production is exhibited by all of the Staphylococci and is the test that distinguishes them from the genus Streptococcus. Although Staphylococcus epidermidis is catalase positive, the entire genus Staphylococcus is catalase positive, and thus this test would not identify the species of the causative agent. Coagulase production is a characteristic of only two medically important organisms, Staphylococcus aureus and Yersinia pestis. Because the organism isolated is described as being non hemolytic, it cannot be S. aureus. Optochin resistance differentiates the major pathogenic alpha hemolytic streptococci. S. pneumoniae is optochin and bile sensitive whereas S. viridans is resistant to both optochin and bile.
-
Question 85 of 100
85. Question
1 pointsA microbiology lab isolates a strain of bacteria that does not possess the recA gene. The process by which this bacterial strain would be capable of performing normally is which one of the following?
Correct
Incorrect
Explanation:
Conjugation involving donation of plasmids from an F+ bacterium to an F- recipient bacterium is the only means of DNA transfer that does not require homologous recombination to occur as a final step of the process. Homologous recombination is a process by which linear pieces of DNA with sequence homology for the recipient cell´s chromosome maybe stably integrated by the action of the recombinase A enzyme (the product of the recA gene). Therefore, a cell which lacks the recA gene would be capable of this form of conjugation, but would not be able to complete any other mechanism of DNA exchange. Generalized transduction is a process of bacterial DNA transfer that involves an accident of the lytic virus life cycle. The transfer must be followed by homologous recombination (requiring recombinase A) if the transferred DNA is to be preserved. Hfr to F- conjugation is a mechanism of genetic exchange that involves the transfer of chromosomal DNA from one cell to another during conjugation. Since it is a linear piece of DNA that is transferred, it must be followed by homologous recombination (requiring recombinase A). Specialized transduction is a mechanism of transfer of DNA from one bacterium to another as the result of an accident during the lysogenic virus life cycle. Since the DNA that is delivered to the new cell is linear in type, the process must be followed by homologous recombination to stabilize any new genes in the recipient. Transformation is the uptake and incorporation of free DNA by competent cells. If any of the newly acquired DNA is to be saved, it must be stabilized using homologous recombination (requiring recombinase A).
-
Question 86 of 100
86. Question
1 pointsAn 18 year boy is taken to the emergency department when he develops a very severe headache accompanied by high fever. He is incoherent and has nuchal rigidity. CSF examination reveals gram negative diplococci. The patient has had five previous episodes of infection with this pathogen, the earliest being at age 6 years. Immunodeficiency related to impaired function of which one of the following should be suspected?
Correct
Incorrect
Explanation:
A wide variety of defects related to complement can occur and can produce immunodeficiency. The pattern shown in the question stem of recurrent Neisseria (gram negative diplococcus) infections is typical of deficiency of complement factors C6, C7, or C8. Other complement deficiencies produce the following patterns: deficiency of C1 q produces a combined immunodeficiency with an SLE-like syndrome; deficiencies of C1rs, C1s, C4, or C2 produce an SLE-like syndrome and glomerulonephritis; deficiencies of C3 or C5 produce a tendency to pyogenic infections. No disease has yet been associated with C9 deficiency. B-cell deficiencies occur in a number of conditions and tend, in general to be associated with an increased susceptibility to pyogenic infections. Eosinophil deficiency is not a usual cause of immunodeficiency. Neutrophil abnormalities can cause a variety of conditions that may present with either pyogenic infections or granulomatous disease. T-cell deficiencies may present with infections with opportunistic organisms such as Candida Pneumocystis, or Cytomegalovirus
-
Question 87 of 100
87. Question
1 pointsA newly developed vaccine protects against respiratory infections caused by adenovirus serotypes 4 and 7. The living non-attenuated virus is delivered in an enteric coated capsule. It establishes an asymptomatic intestinal infection, which stimulates production of memory cells that disseminate throughout the body to protect all the mucosal surfaces from subsequent respiratory attack. The cytokine produced as a result of this vaccination that is most likely to induce a protective response is
Correct
Incorrect
Explanation:
A newly developed vaccine protects against respiratory infections caused by adenovirus serotypes 4 and 7. The living non-attenuated virus is delivered in an enteric coated capsule. It establishes an asymptomatic intestinal infection, which stimulates production of memory cells that disseminate throughout the body to protect all the mucosal surfaces from subsequent respiratory attack. The cytokine produced as a result of this vaccination that is most likely to induce a protective response is
-
Question 88 of 100
88. Question
1 pointsA 44 year old female in her fourth month of gestation with elevated hCG levels. Dilatation and curettage is performed. The products consist of a grapelike mass of tissue composed of thin walled translucent vesicles. No fetal parts are identified. Histologic examination shows hydropic swelling and absence of vascularization of most villi. What is a potential complication of this condition?
Correct
Incorrect
Explanation:
The entity demonstrated in the picture is hydatidiform mole specifically complete mole. Hydatidiform mole is due to abnormal proliferation of trophoblastic tissue. Complete mole results from a process known as androgenesis in which an empty egg is fertilized by a sperm and contains a diploid karyotype. This form will progress to an invasive mole (which infiltrates deeply into the uterine walls) in 10% of cases and to choriocarcinoma in 2% of cases. Partial mole has a triploid karyotype and originates from fertilization of an ovum with one or two sperm. Partial moles are entirely benign and progress to choriocarcinoma in exceptional instances. Histologically, complete mole is characterized by complete absence of vessels within most of chorionic villi and absence of fetal parts. Partial mole, on the other hand, is associated with hydropic degeneration of only a part of chorionic villi and presence of identifiable fetal parts. Carcinoma of the endometrium has no relationship with pregnancy and manifests usually after menopause. Nulliparity, obesity, diabetes mellitus, and any source of unopposed hyperestrinism (whether natural or iatrogenic) are predisposing factors. Eclampsia is a pregnancy related complication that manifests characteristically in the third trimester. It encompasses a clinical syndrome of varying severity from mild cases of hypertension and proteinuria, to severe cases of hypertension, proteinuria, edema, and seizures. It is pathogenetically related to defective maturation of the vasculature of chorionic villi, with resultant placental ischemia and release of vasoconstrictor substances. Sarcoma botryoides has a gross appearance superficially reminiscent of complete mole. This malignant tumor predominantly affects children and may appear as a grapelike mass of tissue that protrudes through the vagina. Neoplastic cells are of skeletal muscle origin (the proper name of the tumor is embryonal rhabdomyosarcoma). On H&E, neoplastic cells appear small and undifferentiated. Yolk sac tumor is a malignant neoplasm of germ cell origin. It develops in children and young adults, either in the ovary or the testis. It is composed of tissue that resembles primitive yolk sac.
-
Question 89 of 100
89. Question
1 pointsA 2 year old child with leukemia develops nephritic syndrome. Light microscopic studies are normal. Electron microscopy shows fusion of epithelial foot processes. The current hypothesis for the pathogenesis of his change is that it is secondary to which one of the following?
Correct
Incorrect
Explanation:
The child is suffering from minimal change disease (lipoid nephrosis), which has a peak incidence at 2-3 years of age. Minimal change disease can be associated with food allergies, medications, or hematologic malignancies, or it can occur, idiopathically. The pathology does not appear to involve complement, immunoglobulins, or immune complex deposition. Rather, an altered cell mediated immunologic response with abnormal secretion of lymphokines by T cells is thought to reduce the production of anions in the glomerular basement membrane thereby increasing the glomerular permeability to plasma albumin through a reduction of electrostatic repulsion. The loss of anionic charges is also thought to favor foot process fusion. Some authors have noted that other conditions associated with T-cell abnormalities such as Hodgkin´s disease and T-cell lymphoma, are sometimes associated with minimal change disease.
Consumption of complement factors is observed in many conditions in which complement activation occurs for example membranoproliferative glomerulonephritis. IgG directed against renal and pulmonary basement membranes is found in Goodpasture´s syndrome a cause of rapidly progressive glomerulonephritis and hemoptysis. Immune complex deposition is associated with type III hypersensitivity reactions including postinfectious glomerulonephritis, lupus nephritis, Henoch-Schonlein purpura, cryoglobulinemia, and bacterial endocarditis. Mesangial IgA deposition is associated with Berger´s disease or IgA nephropathy a cause of glomerulonephritis -
Question 90 of 100
90. Question
1 pointsA 33 year old female has hyperthyroidism every time she gets pregnant. Serum T4 levels markedly increase during the first 10 weeks of the pregnancy, then decline and are only moderately increased during the remainder of the pregnancy. Her condition can best be explained by a mutation in
Correct
Incorrect
Explanation:
The key to answering this question is knowing the plasma concentration profile across the 40 weeks of pregnancy for each of the hormones mentioned. Plasma concentration of human chorionic gonadotropin (hCG) doubles every 2 days during the first 10 weeks of pregnancy and then declines to a level that is one-tenth of the peak for the remainder of the pregnancy. hCG is in the same hormone family as TSH, FSH, and LH. These are all glycoprotein hormones with identical a subunits but different β subunits. Hence, there is a similarity in receptors for these hormones as well. A mutation in the TSH receptor that also made it responsive to hCG would result in increased thyroid hormone secretion during pregnancy due to the increased circulating hCG. Under nonpregnant conditions, thyroid hormone secretion would be normal. This mutation was actually found in a woman who had experienced several early miscarriages. When she was treated for hyperthyroidism with propylthiouracil during her pregnancy, her child was carried to full term. Plasma concentrations of progesterone and estrogen increase throughout pregnancy. By 40 weeks the progesterone concentration may be 200 times greater than pre-pregnancy levels. Estradiol and estrone increase by about 50-told while estriol increases about 11000 fold. While the T3, estrogen and progesterone receptors all originate from the same superfamily, it is unlikely that the woman´s pregnancy associated hyperthyroidism is due to an action of progesterone or estrogen on the-T3 receptor. If this were the case plasma levels of thyroid hormone would be expected to rise throughout pregnancy, with the highest concentration occurring just prior to birth. Furthermore, in the nonpregnant state increases in estrogen during the follicular phase and increases in progesterone during the luteal phase would produce problems in thyroid hormone secretion. This patient has normal thyroid status when not pregnant. Human chorionic somatomammotropin (hCS) increases throughout pregnancy. It is related to the anterior pituita hormones prolactin and growth hormone. A mutation in either the TRH receptor or the TSH receptor would be unlikely to be responsive to hCS because this hormone is unrelated to either TRH or TSH. Furthermore, if such a mutation could occur it would produce maximal thyroid hormone concentration near the end of the pregnancy (not at 10 weeks).
-
Question 91 of 100
91. Question
1 pointsA 55 year old man has higher than normal language output and frequent paraphasic errors. Neurologic evaluation demonstrates that his comprehension of auditory and visual language is severely disturbed. He also has an inability to repeat language. He is suffering from which type of aphasia?
Correct
Incorrect
Explanation:
The question stem describes Wernicke, or receptive, aphasia. It is caused by a lesion in Wernicke´s area, which is located in the posterior part of the superior temporal gyms of the language dominant hemisphere.
Broca (expressive) aphasia is a disorder primarily of language output. Speech is slow and effortful, phrase length is short, and the patient tends to use only common nouns, verbs, and, occasionally, adjectives. Comprehension is relatively intact. Repetition is generally disturbed comparable to the amount of spontaneous output. It is produced by a lesion of Broca´s area, which is in the posterior part of the inferior frontal gyrus of the language dominant hemisphere. Conduction aphasia is produced by a lesion in the arcuate fasciculus, which is in the posterior inferior part of the parietal lobe of the language dominant hemisphere. This effectively disconnects Broca´s area from Wernicke´s area, resulting in the inability to repeat. Global aphasia occurs with the destruction of Broca´s area, Wernicke´s area, and the arcuate fasciculus, resulting in a combination of both Broca and Wernicke aphasias. Mixed transcortical aphasia results from damage to much of both Broca´s and Wernicke´s areas, but leaves the arcuate fasciculus intact. The patient has little spontaneous language output or comprehension but is still able to repeat. -
Question 92 of 100
92. Question
1 pointsA 27 year old woman accidentally inhales a peanut. In the ER it is revealed that the peanut lodged in her right mainstem bronchus. True statement about the blood flowing through the area of lung distal to the peanut is which one of the following?
Correct
Incorrect
Explanation:
This woman sustained an obstruction of a major airway such that the area of the lung distal to the obstruction was perfused, but not ventilated. In this area of lung, there will be no gas exchange, and the V/Q ratio will approach zero. As a result, the PO2 and PCO2 of the pulmonary blood (and alveolar gas) approaches that of venous blood (PO2 = 40 mm Hg, PCO2 = 45 mm Hg). Therefore, the PO2 will be lower, and the PCO2 will be higher than normal. Because the PCO2 is high, pH Al be low. The amount of dissolved oxygen will be lower than nor-mal, not greater. Dissolved oxygen is equal to 0.003 mL 02/100 mL blood times PO2. Because the PO2 is low in this situation, so must be the dissolved oxygen. PCO2 will be elevated and pH will be diminished in this lung area. Both of these factors cause the curve to shift to the right, not to the left. PCO2 will be higher than normal, not lower. PO2 is decreased and approaches that of venous blood, not arterial blood.
-
Question 93 of 100
93. Question
1 pointsAn elderly female in a nursing home has a fainting spell and is brought to the physician. Examination reveals a resting blood pressure of 130/60 mm Hg and a heart rate of 40/min. What is the most likely diagnosis?
Correct
Incorrect
Explanation:
In complete (third degree) heart block, the ventricles beat independently of SA node activity and P waves become completely dissociated from QRS-T complexes. The rate of the ventricular beat is usually 30 to 45 per minute. Because resting cardiac output (CO) is normal and because CO = stroke volume X heart rate, the stroke volume is increased in complete heart block. When the stroke volume increases, a greater amount of blood must be accommodated in the arterial tree with each heartbeat, which causes a greater rise and fall in pressure during systole and diastole. Note that the pulse pressure is 70 mm Hg in this patient (normal pulse pressure is 30 to 50 mm Hg). The pulse pressure is decreased in aortic valve obstruction, cardiac tamponade, heart failure, and mitral valve obstruction.
-
Question 94 of 100
94. Question
1 pointsA 56 year old male has a 3 month history of progressive mental deterioration in the form of memory loss, mood changes, and errors in judgment. His gait is unsteady. No history of seizures, head trauma, or incontinence is present. CT scan and lumbar puncture are unremarkable. Exam reveals hypertonicity of all extremities, bilateral equivocal planter response, ataxic gait, and myoclonic jerks in the lower extremities. The mechanism by which this infectious agent causes its pathology is
Correct
Incorrect
Explanation:
This is the classic presentation of Creutzfeldt-Jacob disease. (CJD).Although the pathogenesis is incompletely understood these patients develop extracellular deposition of abnormal fibrillar proteins in the brain, i.e., amyloid. Autoimmune destruction is not indicated because there is no immunologic response to the deposition of these extracellular proteins; thus there is no chronic inflammation. Although embolization and infarction could complicate the presentation in the age group typically afflicted with CJD these processes are not believed to have any direct role in this pathology. No toxin is produced to account for the presentation in CJD.
-
Question 95 of 100
95. Question
1 pointsA 4 year old girl was initially normal, but began to have painful abdominal bloating and bulky, pale, malodorous stools at about 4 months of age when cereal products were started. Dietary modifications lead to the resolution of the symptoms. The patient is at most increased risk for developing which condition?
Correct
Incorrect
Explanation:
The history strongly suggests gluten sensitive enteropathy (celiac disease), the pathologic hallmark of which is flattening of small intestinal villi. The condition is due to intolerance to gluten, which is found primarily in wheat and rye products. Celiac disease may present in childhood (as illustrated) or in adulthood. Treatment includes complete removal of gluten from the diet. Symptoms can completely remit with dietary modifications. These patients are at increased risk for developing certain malignancies, including intestinal lymphoma and breast cancer.
Inflammatory bowel diseases, in particular ulcerative colitis, carry an increased risk for colon adenocarcinoma. These patients do not have an increased risk of developing Crohn disease or ulcerative colitis. Jejunal strictures can occur in Crohn disease. -
Question 96 of 100
96. Question
1 pointsA 33 year old woman with no prenatal care presents at 39 weeks gestation because of contractions and a loss of fluid. 4 hours later she delivers an apparently healthy girl. Student in the room cuts the umbilical cord and places the placenta in a container. Later the pediatricians become concerned because of uncontrolled bleeding from the umbilical stump. This finding suggests a deficiency of
Correct
Incorrect
Explanation:
Hereditary factor XIII deficiency is an autosomal recessive condition that is unusual among the factor deficiencies in that the presentation is often at birth, when the umbilical stump bleeds excessively, sometimes leading to the neonate´s death. Factor XIII is necessary to stabilize clot formation; in its absence, clots will rapidly lyse. Cutaneous and muscular hematomas are common in affected patients. Bleeding after surgery and trauma can occur, including bleeding into the CNS. Spontaneous abortion in affected women is common. A factor XIII concentrate is available for treatment.
-
Question 97 of 100
97. Question
1 pointsA characteristic feature of primary hyperaldosteronism is which one of the following?
Correct
Incorrect
Explanation:
Primary hyperaldosteronism or Conn´s syndrome is characterised by hypokalaemic hypertension. Patients can present with tetany (alkalosis) and muscle weakness (hypokalaemia). Oedema and oliguria are more features of secondary hyperaldosteronism (cirrhosis); and vitiligo (suggesting auto-immunity) is not a feature.
-
Question 98 of 100
98. Question
1 pointsGaucher´s disease is associated with the deficiency of:
Correct
Incorrect
Explanation:
Hexosaminidase A deficiency is associated with Tay-Sachs disease. Sphingomyelinase deficiency is associated with Niemann-Pick disease. Arylsulphatase-A deficiency is associated with metachromic leucodystrophy.
Iduronidase deficiency is associated with Hurlers syndrome -
Question 99 of 100
99. Question
1 pointsWhich of the following disorders is characterised by an Autosomal dominant mode of inheritance?
Correct
Incorrect
Explanation:
All the others are autosomal recessive of course.
-
Question 100 of 100
100. Question
1 pointsThe patients of a child with cystic fibrosis (CF) consult you wishing to know that what is the risk of their next child being a carrier of the condition.
Which one of the following percentages is the correct risk?Correct
Incorrect
Explanation:
As both parents are carriers for the CF gene then the chances of another child being affected (homozygote) is 1 in 4 (25%).
The chances of their child being free from the CF gene is also 1 in 4 (25%) and the chances of a child being a carrier (heterozygote) is 1in 2 (50%)